Thursday, July 12, 2012

Fallacies – 1. Paper Gold is just like Paper Anything


This is the first in a new series I'll be returning to occasionally. I have recently come across a number of fallacies relating to the subject matter of this blog and my plan is to compile them and then correct them one at a time.

1. Paper Gold is just like Paper Anything

This first fallacy aims to undermine a good deal of what Another and FOA wrote about by claiming that the paper gold market has the same effect on gold as any paper market has on its underlying commodity. This fallacy claims that the same arguments made for an explosive revaluation of physical gold could be made for anything else, therefore they must be wrong.

Commenter "ForLiberty" put it this way:

"This whole 'paper gold is holding the price down' argument makes zero sense to me. It is just a logical nonsense. All paper trades have two parties - bidup and biddown. A paper gold trade could have never taken place if nobody wanted to short it. This is how all markets work. There are paper tomatoes sold too. Is there a conspiracy there too?"

ForLiberty was apparently paraphrasing Martin Armstrong who wrote something remarkably similar:

"They argue that today gold is really paper gold, and the market have multiplied that many times. They argue that the real gold is only about 5 billion ounces. They then argue that the paper gold depresses the price of gold and this is why it is not where it should be right now. All this sound nice, however, you can make the same argument about anything traded today from wheat to stocks and bonds given the derivative markets. Some see conspiracy behind everything."


Is he correct? No he's not. Can we really make the same argument for anything else? No we can't. The paper market for commodities is just as likely to have a levitating effect as a suppressing one because it allows for financial participation by those who have no need or ability to hold the actual commodity. Gold is the only one that is unequivocally suppressed by the existence of a paper market.

No conspiracy. The mere existence of a commodity-like paper market for gold suppresses the price naturally, systemically. Long term systemic suppression of gold is something totally separate and different from short term price manipulation or distortion which can occur in any commodity or paper market.

Here's ANOTHER explaining that the BIS (primarily European central banks at the time) not only anticipated that a paper gold market would lower the price of gold, but that in the 1980s they supported the creation and expansion of this market for that very purpose:

Date: Mon Feb 16 1998 14:40
ANOTHER (THOUGHTS!) ID#60253:

"The BIS leads the creation of a paper gold market that will lower the world price of gold to the extent that it remains above "production costs".

Guess what, it worked! Contrary to all expectations of oil shortages, inflation, debt collapse and what have you, It Worked! But, there is one small problem?

The BIS and other various governments that developed this trade (notice I didn't use conspiracy as it was good business, as the world gained a lot), thought that the paper gold forward market would have allowed the gold industry to expand production some five times over! Don't ask where they got this, as they are the same people that bring us government finance and such."


In other places ANOTHER explains that we should not be upset with the CBs because they were just buying us time. And later he explains what they were buying time for—to make it to the launch of the euro. He also muses about the fact that it's the Westerners playing in this new paper gold market who are most upset about the low price. The physical buyers in the East see it as a gift. But I digress.

Nobody is claiming there are more than 5 billion ounces of paper gold. In fact, there is probably far more physical in the world than paper gold. Enough physical gold to cover all of the paper a few times over perhaps. But that doesn't matter, it is only the flow that matters. It's the same with commodities that get produced and then consumed. It's the flow between production and consumption where the price is discovered in the paper markets. But gold doesn't get consumed at a rate anywhere close to its next closest competitor. It just accumulates.

In commodities the paper market regulates the flow between the producers and consumers, acting as a kind of a shock absorber against unexpected supply and demand shocks. But gold is different because it just accumulates. There are two main differences between gold and everything else. The first is that gold just accumulates rather than getting consumed, so there is no reason for there to ever be a supply side shock, even if all the mines suddenly stopped producing. In fact, today we have a 60 year "supply overhang" in gold. Nothing else comes close.


The second difference is that the vast majority of demand for gold is in currency terms, not weight terms. This is not true for commodities. If you need a ton of copper for a construction site, you need a ton of copper. That's weight-denominated demand. But gold demand is overwhelmingly in currency terms. If you need a tonne of gold, what you really need is $50,000,000 worth of gold. It doesn't matter how much it weighs because you're just going to stick it in a vault.

Having a paper market as a shock absorber for the gold market only has the effect of keeping the price too low. My explanation for the LBMA survey discrepancy is a perfect example.

Since gold is not consumed by consumers or industry the way corn, oil, copper and grains are, and because it simply accumulates, supply shocks are not economically critical. On the demand side, gold is apparently used as a "safe haven currency". And we apparently had a demand shock of around 7,575 tonnes in Q1 2011. The normal supply for that period would have been around 700-1000 tonnes, so the paper gold market acted as a shock absorber and absorbed that demand shock by expanding. That way the price of gold only rose $30 in a quarter with a demand shock of 10 times the normal physical supply flow.

But that wasn't really demand for 7,575 tonnes of gold. It was demand for $337B worth of gold. Hypothetically, if the price of gold had been $55,000/oz. in Q1 2011, that demand would still have been for $337B worth of gold, the only difference being that the $337B demand could have been supplied by only 190 tonnes (a mild 20% increase in flow rather than an extreme 1,000% increase) and the price of gold would therefore have barely felt a bump in the road, even without a paper market shock absorber.

Therefore, having an elastic paper market shock absorber for gold is only necessary if the price is too low, because there will always be plenty of supply if the price is high enough (60 year supply overhang, remember?). At today's price, having a paper market shock absorber is apparently necessary to keep the gold market from blowing up.

It logically follows that it is the very existence of the paper gold market which is keeping the price too low, because if you took it away, price alone would have to regulate the flow. Take the paper market away from other commodities and you simply remove the investor/speculator money in the middle thereby exposing producers (and consumers) to unpleasant shocks.

We have no idea what the "stock" of paper gold is. The LBMA survey only gave us a glimpse of the flow (paper gold turnover) over a given time period (Q1 2011) and in a given market (loco London spot, forwards, options and swaps, with spot transactions being 90% of the reported trades). That turnover was 2,700 "tonnes" of paper gold per day with 64% of the LBMA members reporting. We only got a lucky glimpse because the largest banks in the world (bullion banks like JP Morgan Chase, Goldman Sachs, HSBC, Barclays, Deutsche Bank, Credit Suisse and UBS) are lobbying for a technical rule change that will make their overall Basel III compliance easier.

Sincerely,
FOFOA

616 comments:

«Oldest   ‹Older   401 – 600 of 616   Newer›   Newest»
burningfiat said...

AD was pretty on point though, r we gonna worry about purple unicorns flying out of our butts too?

lol no, but contemplating a society with no energy or resource scarcity could turn on a light bulb or two regarding the role of energy in our _current monetary system_ or the _freegold system_. Potentially beneficial!

Motley Fool said...

JR

1) If the US had sole access to this technology initially, then yes. This extension does move us further into the realm of implausibility though.

2) We agree that the demise of the $IMFS is certain, and that just the timing is uncertain. Why extend? Because gradual replacement is less traumatic than economic collapse.

Advanced knowledge of science too eh. Aren't you just a polymath. :P

For amusement value, I've seen an interesting conceptual solution to the source charge problem, which would allow for 'free' energy. Additionally this opens up a new field of nuclear reactions which might include transmutation from a baser metal into gold.

...

"When a distinguished but elderly scientist states that something is possible, he is almost certainly right. When he states that something is impossible, he is very probably wrong."

Arthur C. Clarke

TF

Anonymous said...

Just for your information.

GLD lost more than 250000 ounces today. This is yet another buy signal in my trading strategy although not enough for a Lance Lewis puke. There was no other signal in the meantime.

GLD inventory is now less than immediately after the May 22 puke.

Victor

ForLiberty said...

@FOFOA - thank you for the lengthy response. I don't want to get in an aggravated debate because I appreciate your answers, but I think I should at least make a couple of points to acknowledge your answer.

1. Your argument that gold is different than everything else for it has such huge above ground supply that no supply shock is possible, that is a very weak argument and to me it's immediately obvious that it's false. Mining and above ground supply have very little bearing on the market supply - the flow - as you like to call it. You need sellers all the time and it is very possible that supply is reduced even with increased mining. You yourself quote another - gold is funny in that way - it disapears as price goes up. Sorry for not quoting exactly but can't dig to get that.

2. you never addressed the problem of each transaction having a seller of gold, for the exact same amount at the same price as the buyer's bid. That is a terminal problem with the 'hold the price down' theory. It can never be really resolved, and blurry sophistry is usually used to go around this problem and hang some straw man. What would happen to those sellers in a paperless market? Some of them may own physical and sell the physical. Some of them may drop out of the supply but there will be their mirrors who will drop out of the demand because they only wanted a paper transaction.

What matters, Mr Fofoa, is what is the total quantity that all buyers want to buy at each price point, and what is the total quantity that all sellers want to sell.That determines the supply/demand creation of the price. whether the parties chose to do the transaction on paper or with physical exchange is irrelevant to the total demand/supply.

You are in the same boat with the european governments trying to ban naked shorts of bonds and even stocks during a slump. You all are in the same boat. When it comes to your item, bonds or gold, you believe the naked shorts have an impact on the market price when they DONT.

JR said...

Why extend? Because gradual replacement is less traumatic than economic collapse.

Why are you talking about economic collapse, I though we were talking about "free energy" that would allow the US to pay its debts in real terms?

There is no need for political or structural for support the dollar in such a scenario, so the whole reason why the $IMFS exists (because the ROW found the $IMFS more palatable than a global transactional currency reserve void) is moot.
Why would anybody keep saving in it going forward, unless we stipulate the US is the only one with the power.

=========

Free energy does not = the absence of scarcity, its just means greater growth.

JR said...

Hi ForLiberty,

I agree your response makes clear you have no idea what FOFOA is writing about.

Cheers, J.R.

burningfiat said...

JR,

Free energy does not = the absence of scarcity, its just means greater growth.

Yeah ok, see your point. My point was that the discontinuum (to the upside) in resource availability would be felt equivalent to absence of scarcity.

Example: So where it before was only economical to extract z tons of aluminum. Now you can extract 1000 x z tons economically. And also with free energy, you can recycle as much material as you would like (and as fast as you care to).
The only time you hit the resource ceiling with free energy is when you need more of a material in circulation in the economy at any point in time than there exist on your planet (or solar system). But point taken: Given enough mind-boggling exponential growth, you'll eventually hit that ceiling for a number of resources...

To go further down the purple unicorn road: With free energy already, the next law of physics that ultimately stops economic growth is the speed of light. The barrier at which you can't transport material to their place of use fast enough, no matter how much energy is put into acceleration the objects. #relativitySucks

===========

ForLiberty,

I think you're confusing short selling and naked short selling.

Motley Fool said...

JR

"Why give US exprbitant provilege?"

That's why. Given greater growth rates, the pain of continual declining support may be bearable in order to smooth a transition away from the $IMFS. Something we have seen regardless.

TF

Ps. Free energy + star trek like replicators = lack of scarcity. :P

enough said...

VTC,

I noticed that GLD has been coughing up 2-3 tonnes on quite a few days over the last couple of weeks with zero inflow days over that same period.....

Anonymous said...

JR said...

"Free energy does not = the absence of scarcity, it just means greater growth."

I concur! The world will keep turning just the same, only a little faster perhaps;) It's only the natural evolution of "things", which is expected... at least by some.


Burning,

I've had my eye on LENR for a few years now. They're advancing to the stage where it is practical and, in fact, a reality. There's a few parties making advancements with it. If all goes as planned I will have a unit to test for myself.

My other eyeball has been on the Noble Gas Engine... my little pet:)

Just In Case You're Wondering

JR said...

Given greater growth rates, the pain of continual declining support may be bearable in order to smooth a transition away from the $IMFS.

There is no need for political or structural for support the dollar in such a scenario, so the whole reason why the $IMFS exists (because the ROW found the $IMFS more palatable than a global transactional currency reserve void) is moot.

Ps. Free energy + star trek like replicators = lack of scarcity. :P

Free energy does not = the absence of scarcity, its just means greater growth.

More wealth does not = absence of scarcity

Nickelsaver said...

TF,

Free energy is a misnomer. Clean energy, renewable energy, efficient energy are perhaps better descriptions. For what could be more free than extracting natural resources from the ground using old technology, as opposed to generating energy via advanced technology that doesn't even exist.

Also, this seems like a ridiculous dream when you consider that we can't even get away from this.

But if we are ever to approach anything like that. It is not going to come about via the current political landscape. We can send a man to the moon, but we can't produce a car that runs on Nat gas or electricity?

Victory said...
This comment has been removed by the author.
Victory said...

Date Tonnes
28-Jun-12 1,281.62
29-Jun-12 1,279.51
2-Jul-12 1,279.51
3-Jul-12 1,279.51
4-Jul-12 HOLIDAY
5-Jul-12 1,279.51
6-Jul-12 1,279.08
9-Jul-12 1,275.46
10-Jul-12 1,271.24
11-Jul-12 1,271.24
12-Jul-12 1,269.73
13-Jul-12 1,269.73
16-Jul-12 1,266.11
17-Jul-12 1,266.11
18-Jul-12 1,266.11
19-Jul-12 1,257.05

Not a one day puke but there has been a steady drain of 2% (24 tones) since June 28th out of GLD. Significance?

Also interesting interview with Jim Rickards, the very last few seconds of the interview Jim says BIS involved with gold price manipulation/management

-v

Beer Holiday said...

"In this house, we obey the laws of thermodynamics!" - Homer Simpson

Who was taking bets on free energy arriving soon? Where do I sign up.

Free energy = perpetual motion machines in my book, do we have different definitions? If not I'll meet you at the All Inn.

Do you mean cold fusion (not free energy, obeys thermodynamics)

If you mean hot-fusion, or improved fission fair enough, but a smaller bet is still on because I don't think Gov's today have the ability to run fission properly, let alone improve it.

The credibility problem comes from the last time cold fusion was unfortunately pushed by those who let wishful thinking overcome scientific principles.

I can't prove cold fusion doesn't work, and I know some smart folks who think it might (they tend to save that talk for the pub however). I suspect it doesn't because no-one's seen it yet repeatably. That includes Astro observations, and there is some pretty wacky stuff out there, neutron stars, black holes, our sun like most is a giant hot fusion reactor).

Number of cold fusion stars seen to date: zero. Number stars in observable universe: ~10^24 . They would mostly be hot fusion I guess.

Michael H said...

Regarding 'free energy':

Richard Heinberg made the point that the discovery of oil was essentially the discovery of 'free energy' relative to what was available at the time. Look what society has done with that energy.

I agree with JR, free energy does not mean free everything. The resource limit would just move down to the next limiting factor.

ForLiberty,

"1. Your argument that gold is different than everything else for it has such huge above ground supply that no supply shock is possible, that is a very weak argument and to me it's immediately obvious that it's false. Mining and above ground supply have very little bearing on the market supply - the flow - as you like to call it. You need sellers all the time and it is very possible that supply is reduced even with increased mining. You yourself quote another - gold is funny in that way - it disapears as price goes up. Sorry for not quoting exactly but can't dig to get that."

This is idiotic and makes no sense.

"immediately obvious that it is false" What else has a 60-year above ground supply?

The point is that there is *always* more supply available at the right price. It's not like, say, palladium, where if someone buys up all the mine supply and convinces the refiners to "take inventory" at an inopportune time, the consumers of the metal have no alternative source to turn to.

As for the "gold is funny in that way - it disapears as price goes up" -- see FOFOA's point that demand for gold is in currency terms. So 'disappears as price goes up' means 'less flow by weight as the price goes up' i.e. same flow in currency terms.

What you are not understanding is the difference between the paper gold market and other paper commodity markets. FOFOA is not talking about COMEX shorts etc. He is talking about the London gold market where one buys unallocated gold -- credit denominated in gold ounce.

I'll repeat: credit denominated in gold ounces.

Can you get credit denominated in bushels of corn? Unallocated, fractionally reserved corn accounts at your local grain elevator?

Think of it in terms of currency. What would the general price level be if the only currency available was physical bills, with no bank accounts, credit cards, checks, etc?

Michael H said...

"2. you never addressed the problem of each transaction having a seller of gold, for the exact same amount at the same price as the buyer's bid. That is a terminal problem with the 'hold the price down' theory. It can never be really resolved, and blurry sophistry is usually used to go around this problem and hang some straw man. What would happen to those sellers in a paperless market? Some of them may own physical and sell the physical. Some of them may drop out of the supply but there will be their mirrors who will drop out of the demand because they only wanted a paper transaction."

I started addressing this in my previous comment.

"each transaction having a seller of gold, for the exact same amount at the same price as the buyer's bid."

Right, but the seller has no gold, only gold liabilities to sell, hence fractionally-reserved unallocated. However, for such accounts, the buyer is fully paid up, not buying on margin like on COMEX.

"What matters, Mr Fofoa, is what is the total quantity that all buyers want to buy at each price point, and what is the total quantity that all sellers want to sell.That determines the supply/demand creation of the price. whether the parties chose to do the transaction on paper or with physical exchange is irrelevant to the total demand/supply."

Thank you for the econ 101 lesson. Now let's look at econ 102: if you offer credit denominated in something, then the value of that something is decreased because you have synthetically increased its supply.

"You are in the same boat with the european governments trying to ban naked shorts of bonds and even stocks during a slump. You all are in the same boat. When it comes to your item, bonds or gold, you believe the naked shorts have an impact on the market price when they DONT."

You are talking out of your ass.

FOFOA is not 'trying' to do anything, he is describing how things work.

You think the naked shorting of, say, 7500 tonnes of gold would have no impact on the price?

Beer Holiday said...

I might be helpful to define some completely separate and different concepts:

Free energy: Sources of energy that don't obey the second law.

Clean energy: Solar and solar derived sources.

New energy: Better fission, hot fusion,

Awesome energy: Cold fusion.

Oil: Black gold, Texus tea.

Beer Holiday said...

Oil is even better than many other cheap energy sources, it's pre-stored energy that you can put into supertankers and take were you want. Even if free energy cropped up tomorrow, you still have the power transmission problem.

Aiionwatha's Nation said...

It's a done deal deal. Signed sealed and delivered, the only question left is what will be the remaining power structure.

The Euro was a fall back plan, didn't happen. Now we have door #2, starve like the planters that supported your sorry asses for the last 30 years after we lost the ability to divvy up their production through force and divert their surplus through financed trade. We tried, but they have had enough so there just isn't enough to go around. Deal with it.

As I recall, once again, debtor is servant to the lender and they can peel back that banana much slower than they can the current $IMFS. Good news is 99% of the folks are eyeballs deep by definition so it should be fairly easy to manage a few % at a time with slow process deflation.

Oh, you withdrew your giant account did you? Never happened. Might be some left later if you keep still.

Welcome to the machine.

As far as the GLD goes, nobody knows what these card sharks are playing for but the landscape has changed.

Anonymous said...

Beer Holiday,

"Even if free energy cropped up tomorrow, you still have the power transmission problem"

What's the problem? The infrastructure is already in place. Wires are wires.

Interestingly, everything is made of energy, you can call them particles or maybe strings? Everything... Only different expressions of the same "stuff". It's just that some forms of "stuff" are easier to liberate usable energy from. We have a lot to learn yet.

I'm just here for the ride, no need to place any bets:) Or, god forbid, no Breaking The Law

Beer Holiday said...

SV, great link, I haven't seen that in ages.

Mostly I was hoping to delineate the discussion. If "free energy" = new solar tech, you have a grid and storage problem.

Your term for free energy is the same as mine, but you still have grid (minor) issues. Our grid is designed like a river to funnel hydro, coal power etc. So you need to change the grid once you start breaking the 2nd law thermo with impunity :-)

Also not nearly as much copper wire grid in e.g Africa and much of the developing world. Oil and energy storage is still vital.

On your energy philosophy question: my position is there is no energy in nature - energy is just the number we use to describe nature. There are no right/wrong answers, just thought you might be interested.

costata said...

I came across this quote today and I thought it might be food for thought (my emphasis).

Researchers at JP Morgan Asset Management have recently pointed out that cash accounts of US investors have been increasing at a "remarkable" rate and this is "despite historically low interest rates and negative real yields".

These cash balances in the US now stand at $US10 trillion ... and exceed the total mortgage debt of American households.


http://whitesifm.com.au/wp-content/files/Aust010512.pdf

In the article (from May 1st, 2012) a graph shows the trend in both mortgage debt and cash balances. I'm wondering if there is something important to be gleaned from this graph.

The correlation between the amount of cash balances and RE mortgage debt is close all the way back to 1959 and perhaps even earlier looking at the consistency over such a lengthy period. And the trend in mortgage debt outstanding is currently declining while the trend line in cash balances has crossed over and it is rising.

What, if anything, is this graph trying to tell us? (BTW this isn't a rhetorical question.)

costata said...

I just had another thought but I'm too tired right now to connect the dots (if there are any dots to be connected in the first place).

That thought was about the reported US dollar shortage in China and suggestions that private interests in China were short up to $700 billion in the dollar. Coupled with the reports that private money may be on the move out of China prompts me to ask:

Has the big private money in China found ways to bring some of those orphan dollars home to pappy in the USA?

(Again, this is not a rhetorical question.)

DP said...

@costata,

What, if anything, is this graph trying to tell us?

That when one person takes on a debt and spends the borrowed cash, someone else takes on a cash balance. (They may well then spend it somehow, transferring the cash to someone else's account… etc.)

That demand for US mortgage credit peaked around 2006-2007.

The correlation between cash accounts and mortgage debt seems to be tight, perhaps indicating that (until 2009 when the two lines become distinctly uncorrelated and remain so) the dominant source of new money was mortgages — but since then it's something else.

http://www.youtube.com/watch?v=EvZifMibH38

Jeff said...

Price drop coincides with GLD inventory drop. Time to raise the price and replenish the stock?

AdvocatusDiaboli said...

lets get back to the topic, shall we:
FOFOA claimed:
The gold paper market is different from other paper markets, because it does not consider the "stacked physical" gold.....(then he goes on with some claims, let's just forget for a moment if those reasonings are valid or not)....and arrives to:

It logically follows that it is the very existence of the paper gold market which is keeping the price too low

I thing this is the essential statement of this FOFOA post. Now the only appropriate question under such assumption/claim to pose is:
why never plot a realistic market of the future which would covers this essential properity of the asset gold, taking the above ground stacked gold into consideration?

Since none of the freegolders adressed that issue and probably FOFOA is also aware of that issue since the very beginning of FG, the only conclusion possible to draw is: Not possible, the market is what it is. You can have plenty of gold whereever you look at the paper price of gold, proves to me that the paper market price discovery works just fine.
Greets, AD

AdvocatusDiaboli said...

or to precise my question: How would a gold market have to look like, to arrive at those fantasy prices, that FOFOA would consider not "suppressed" or unappropriate?

Michael H said...

AD,

"or to precise my question: How would a gold market have to look like, to arrive at those fantasy prices, that FOFOA would consider not "suppressed" or unappropriate?"

The key is that the demand side of the market will change.

Right now there is enough gold for everyone who wants it at this price, because demand is not particulary high.

"The big change" that is expected is the demolition of the credibility of debt as a store of value. When that occurs every person, entity, and country who is looking to save a surplus will take a good long look at saving in gold.

Keep in mind that unallocated gold accounts at the BBs are debt as well -- liabilities of the BB -- and so such accounts will also lose credibility.

Aaron said...

And here comes the Village Idiot again with more non-sense.

AD, it’s pretty clear your only goal is to twist information and build straw men to confuse new readers at this blog. And it’s also obvious to the rest of us you don’t have the slightest clue about what Freegold is.

If you are not busy this afternoon, perhaps you could do us all of a favor and take a leap off of the Emmerich Rhine and take your buddy Alien with you.

AdvocatusDiaboli said...

Michael,
Yes, you got it:
Right now there is enough gold for everyone who wants it at this price, because demand is not particulary high.
Absolutely, nobody wants those useless yellow stones. That's the fact, nothing else. Absolutely what I am saying, the market works. Nothing is suppressed due to "paper".

AdvocatusDiaboli said...

Aaron,
thanks for that feedback, now I am much wiser, although you didnt touch that fair question, not even lightyears close.

Michael H said...

AD,

Let me be more precise:

Right now there is enough physical gold for everyone who wants it at this price, because demand for physical gold is not particulary high, thanks to the fact that gold-denominated credit trades at par with physical gold.

DP said...

AD: Absolutely, nobody wants those useless yellow stones. That's the fact, nothing else. Absolutely what I am saying, the market works. Nothing is suppressed due to "paper".

ORLY? Ya think?

MH: "The big change" that is expected is the demolition of the credibility of debt as a store of value. When that occurs every person, entity, and country who is looking to save a surplus will take a good long look at saving in gold.

Keep in mind that unallocated gold accounts at the BBs are debt as well -- liabilities of the BB -- and so such accounts will also lose credibility.


Moar MH: Right now there is enough physical gold for everyone who wants it at this price, because demand for physical gold is not particulary high, thanks to the fact that gold-denominated credit trades at par with physical gold.

#YAAaay MH!

AdvocatusDiaboli said...

Michael,
although you didnt adress my original question:
So what, physical=paper. Persons who want paper for paper, get paper for paper. Persons who want physical for paper get physical for paper. Isnt that the whole purpose of a market place?

About your "credit" argument: Isnt it more the opposite? If I want e.g. next year physical copper therefore I buy today paper copper, what's the difference with gold?
Yep, with copper there might be a delivery problem, no matter how high the price is, since no "stock" available, but still I need my copper. With gold on the other hand, lots of possible holder that might part with there gold if price is high enough. And lots of people that are happy to be cashed out with paper settlement.

So how is the paper gold market not appropriate, or how should it look like to be appropriate?
Greets, AD

AdvocatusDiaboli said...

DP,
you are speaking of some prediction. Shall those be right or wrong, who knows.
Remember the silver folks telling everybody at $50 that the paper price might fall to $25, but than you would not get any? Isnt silver also part of the BB?
Okay, but I was asking about the market place, any suggestions, how it would have to look like that it would not "suppress" the price in the eyes of Freegolders?
Greets, AD

Michael H said...

AD,

"About your "credit" argument: Isnt it more the opposite? If I want e.g. next year physical copper therefore I buy today paper copper, what's the difference with gold?"

There's a difference between copper and gold.

Yes, people buy copper paper now because they want delivery of physical copper next year.

But people buy gold now because they want to be able to sell it next year (or next decade), not just for currency but for real goods and services.

There is no 'credit copper', only copper futures. There is a difference. The former is fully paid up, supposedly accessible any time on demand. The later is a contract for delivery at a specific future date, entered into by paying margin collateral only.

"With gold on the other hand, lots of possible holder that might part with there gold if price is high enough."

Exactly!

"And lots of people that are happy to be cashed out with paper settlement."

For today, sure, since paper settlement will still give adequate returns of real goods and services.

But if at some point in the future you were a holder of BB gold liabilities as your savings, and those liabilities lost credibility to deliver, then you would not be so happy with your paper settlement, since it would only be for pennies on the dollar.

Jeff said...

Gold is still cornered.

FOFOA: the FOFOA view is that the system itself is, and always has been, the culprit. And that the bullion banking system must and will revert to a non-fractional, non-lending, 100% reserve banking system. Not the fiat banks. Just the Bullion Banks. The CBs demand this, as Another told us a long time ago, because physical gold is cornered by real wealth at these prices, and they (the CBs) will not give up any more of theirs...

And that is how the banks are using this "vaulted cash" to delay the revelation that a bank run is already fully underway. They are slowly buying back those "special accounts" in order to move that cash, a little at a time, into safety deposit boxes for the big customers that are actually "running on the bank." As long as no one runs out of the bank's front door yelling "the bank is out of cash," then the run hasn't reached the panic stage yet. But that doesn't mean it isn't happening.

We cannot know the actual state of the BBs' books from what is visible for analysis. So how fast could all of the physical gold reserves be spoken for? As frightening as it sounds, worst case, they may already be. When I think about Jim Rickards' second-hand account combined with the fact that someone is draining GLD, it seems like we could be in the final stage of "extend and pretend that there is not a run on the bullion bank reserves."

What we have here is an explosion in the bullion banks' physical leverage factor, not through an increase in lending this time (the lending is actually declining), but through customer withdrawal of reserves, with no physical backstop. Even a bank with a conservative leverage factor can experience a bank-busting, system-crashing run. Public confidence is the only thing that stands in the way. This is how a classic bank run runs.

Jeff said...

The paper discusses a proposal to mitigate the vulnerability of MMFs to runs by introducing a “minimum balance at risk” (MBR) that that would provide a disincentive to withdraw funds from a troubled money fund. The MBR would be a small fraction of each shareholder’s recent balances that would be set aside in the event that they withdrew from the fund. Most regular transactions in the fund would continue as before, but redemptions of the MBR would be delayed for thirty days. The delay would ensure that redeeming investors remain partially invested in the fund long enough to share in any imminent portfolio losses or costs arising from their redemptions.


http://www.newyorkfed.org/newsevents/news/research/2012/rp120719.html

AdvocatusDiaboli said...

Michael,
either you are contradicting yourself, or you consider other people completely stupid:

"But people buy gold now because they want to be able to sell it next year (or next decade), not just for currency but for real goods and services."
you can not pay with gold for real goods directly, remember? At least not as long the world has not burnd to the ground or you happen to live in some third world nation.
...
"But if at some point in the future you were a holder of BB gold liabilities as your savings,"

you see the flaw? You are assuming that those "giant" gold holders blindly trust in paper gold, but the very smart mummy basements keyboard commando Crash-JPM-conspiratists know it better.

Okay, but I was asking about the market place, any suggestions, how it would have to look like, that it would not "suppress" the price in the eyes of Freegolders?
that alone is the only topic of this post of FOFOA.

DP said...

Like this, initially?

AdvocatusDiaboli said...

DP,
what you mention is more or less a question of settlement of these "supposingly naked short positions" (but most probably of paper gold holders, that never ever in their lifetime intended to even look at their useless yellow stones), but no answer to:

How would a gold market have to look like, to arrive at those fantasy prices, that FOFOA would consider not "suppressed" or unappropriate?
Greets, AD

Motley Fool said...

AD

It's a simple enough answer. No paper speculation on the price of gold encouraged, ie. no legal enforcements of such contracts in specie.

"You can have plenty of gold whereever you look at the paper price of gold, proves to me that the paper market price discovery works just fine."

Really now? Can china? They have a stated goal of acquiring at least 10k tonnes. Can they do so without issue immediately? It's not like gold is being consumed, so it does exist as a stock somewhere. No industrial shortage would prevent it. How about monetary reserves? Is that what is stopping them? They don't have enough spare dollars to buy the gold?

Stop spouting such infantile bullshit please.

TF

AdvocatusDiaboli said...

"Can china? They have a stated goal of acquiring at least 10k tonnes."
who? The PBC? Some private bank? Some jewelery? I love it, when this magical "they" starts popping up.

And if "they" wanna buy, why are "they" no buying at such supposingly much too low "supressed" prices? Worldwide lots and lots huge amount of LBMA/retail gold to get, looks more to me, "they" are even starting to take any physical at all, not even outside the "evil suppressing" paper market.

As you would say: Stop spouting such infantile bullshit please.

Motley Fool said...

AD

Let those who have eyes open them to see.

http://www.zerohedge.com/news/china-imports-more-gold-hong-kong-five-months-all-uk-combined-gold-holdings

I'd say they are giving it the old college try.

TF

Ps. And yes, that is a stated goal of the PBOC.

AdvocatusDiaboli said...

and if you say, "well but 10kt..."
face it: since those "holy prediction" of that jerk 13yrs ago, more than 32kt have been mined, still counting up.

Jeff said...

They have a bath salt problem in Germany?

Someone is probably very happy to get 32kt the past few years. Who thinks they can repeat that feat?

Anonymous said...

Beer Holiday,

my position is there is no energy in nature - energy is just the number we use to describe nature.

Energy is not directly observable, but it doesn't mean that it doesn't exist. LOL.

Basic physics is Law of conservation of energy and there is no exception to this law. To this day, it has never been violated.

Energy is a number that is used to describe the ability of a system to do work, doesn't necessarily have to be nature alone.

AdvocatusDiaboli said...

Yes, we do have a severe bath salt shortage here in germany, therefore still hanging to the question:
How would a gold market have to look like, to arrive at those fantasy prices, that FOFOA would consider not "suppressed" or unappropriate?

since today FOFOA claims:

It logically follows that it is the very existence of the paper gold market which is keeping the price too low

maybe once I get my bath salt, I will stop questioning and just join the chorus "its suppressed".

DP said...

I suspect they never ever in their lifetime intended to even look at their useless yellow stones, because they sincerely believed the bank was keeping "their gold" safe on their behalf.

A bit like our friends who believe in the banks keeping "their money" in the vault for whenever they want to "take it back out". Also the tooth fairy.

"After all, you only find out who is swimming naked when the tide goes out."

Michael H said...

AD,

"what you mention is more or less a question of settlement of these "supposingly naked short positions" (but most probably of paper gold holders, that never ever in their lifetime intended to even look at their useless yellow stones)"

You keep looking at the market today and thinking it will go on like this forever.

After the buyers of paper gold lose their entire investment, do you think others will be so quick to continue buying this paper, or will they finally decide that it would be a good idea to get their hands on some useless yellow stones, to make sure their ownership is unambiguous?

"How would a gold market have to look like, to arrive at those fantasy prices, that FOFOA would consider not "suppressed" or unappropriate?"

Let me repeat what I wrote above:

"The big change" that is expected is the demolition of the credibility of debt as a store of value.

So the gold market would look like this:

Only unambigious claims on distinct pieces of physical gold are worth anything. There is no gold-denominated credit.

DP said...

FOA (09/03/00; 16:27:20MD - usagold.com msg#34)
Of Currency Wars


Hello ALL!

Let's hit the trail!

While we walk view to the southwest:

Is Gresham's law no longer a law? You know, the one that say's good money will drive out bad money. One may look at our modern gold markets and say "help, it's not working". Indeed, if gold is the good money and dollars are the bad money, how come gold has gone down for all these recent years? Worse yet,,,,,, aside from price considerations, gold does not seem to drive out the bad money as people accumulate more of the yellow stuff!

Well, in truth gold can't compete because our modern world of gold mostly consists of the same bad money paper that currencies are made of. That's right, this modern world of paper gold very much functions as the same IOUs that currency is based on,,,,,, not much different from fractional reserve dollars.

So this is the same failing gold markets and resulting prices we all hear and read about. The same markets gold bugs watch with increasing despair as Gresham's law works it's will. By saying "paper gold cannot drive out paper currency" this law is proving that hard money investors have hitched their wagon to the wrong horse. A paper horse! Today, more than ever, investing in an industry or paper vehicle that requires a functioning gold banking (paper) intermediary will be devastating as this modern gold market evaporates.

Make no mistake, physical gold already contains many times the value these paper markets have established for it. When the price of physical traded gold runs far away from this imploding paper gold arena, the value of real money will them truly reflect where our bad money exists today.
Obviously, we are talking about dollars.

True, gold will reset itself in value compared to all world fiat currencies. But, that percentage reset will be viewed in a different context than when gold money was ordained by governments. Gresham's understanding applied more to gold as a bankable currency, not an asset holding "in and of itself". This is the future of "freegold" in our time. It will be much like comparing an advancing stock to the currency it's denominated in, a rising asset,,,, not a competing money!

Now look northeast, into the valley:

What will make this "modern gold market evaporate"? Well, value in a paper contract is a funny thing,,,,,,, it can change radically when no market bids exists for it to trade in. Like paper dollars, contracts have no value without a trading market demand. Walk into any store,,,,, if everything is suddenly priced in a physical trade format, our dollars suddenly become worthless, no?

If the gold market was to shift to say, 5 day hard delivery, how could one trade their contracts for gold? Yes, you guessed it, paper would trade all right,,,,, at a huge discount. But in short order, as a spiking price lunges upward into the thousands,,,, and doesn't come back to earth,,,,,, what counter party on the other side of your contract could deliver? Further, how could the bullion banking system match liabilities and make good on a cascading default?

DP said...

Stop here and see how it could happen two ways (or a combination of both):

You see, all it takes is for one or two government and/or private entities to pull the cord. Most all of you long ago came to the same conclusion; a Dollar / Euro currency dispute could set this off. Outside parties begin buying gold with dollar reserves,,,,, on the barrel head for 5 day placement. It begins with twenty or thirty 100 ton orders ,,,,,, a billion$$ or so each! Not derivative orders, mind you,,,, hard delivery orders that aggravate and outline the soft nature of modern gold banking. They keep coming,,, days on end! Then, suddenly the paper markets "are no more".

OR:

The price of oil rises until price inflation can no longer be contained. Unmined physical gold is withheld from the markets to such an extent that even limited demand runs the physical price to a large premium. More and more investors pay a larger escalating premium to get physical "now". Such a premium overwhelms and discredits the function of paper market pricing. Bullion Banking must revert to currency banking to cancel out it's contracts. The run begins.

Let's check our political map to see where we are on the trail:

The wonderful recent essays by Mr. Howe expose and document a system as it currently stands,,,,at the end of it's timeline of usefulness. The purpose, need and use of gold banking has run headlong into a world class political storm. This end time battle has been in the making from before Another ever started writing. Truly, this Gold War will be about a transition from world dollar dominance.

As I stated in an earlier USAGOLD post (listed as #35569):

----- Today, oil flow has moved from playing a fundamental game of pricing "use value" with supply and demand to pricing it's "monetary value" in supporting any major currency block. Concessions are now there for the taking by oil producers. Dollar prices for oil can rise considerably higher with the US giving behind the scene support for this action. In addition, the world paper gold markets can and are being dismantled as a further concession to retain dollar settlement of oil. -------------

It's easy to see today that most of the major world oil reserves have had their value politically converted by the Euro's successful birth. The current trading value of the Euro is a small factor compared to this "existence" worth in our political currency wars. This Euro has broken the "cheap dollar" value placed on oil by our one currency world and now allows dollar oil prices to soar without constraint. Oil has indeed been rising after the Euro's first few months of existance The prize of this master play on our Chess Board ,,,,? This new pricing thrust, unlike early OPEC drives of the 70s and 80s, will crush the modern paper gold banking game and place the dollar and Euro on a level playing field.

And so the contest begins for real,,,, no longer must we talk in a future mode,,,,,May the best man win!

Thank you
FOA/ Your Trail Guide

DP said...

FOA (09/04/00; 20:23:42MD - usagold.com msg#35)
Correction?


Hello again,

On the Veranda for an explanation:

I have a clarification to make concerning our observations during the last hike. Right after it was done, a USAGOLD poster with the handle Mr. Gresham (#35943) pointed out my backward way of presenting Gresham's law. Well, he was very right. My post had a way of reversing the concept.

I thought it would be understood, but after reading it again it was obvious that many would walk away with the wrong impression.

Gresham's law say's that "bad money drives good money out of circulation". In our context, on the trail it was presented from a position that "good money drives bad money into circulation". No, I didn't say that outright, but that was my mindset. You say, you can't read my mind? (smile)

We used this way of presenting the situation because it better illustrated how gold today, has not risen in value against dollars (the bad money). By failing in this process, the stage for a value judgment to take place where people hold gold (the good appreciating money) and spend dollars (the bad money) could not happen. Paper gold prices were not driving dollars into circulation! There is no FreeGold price to judge and convict our fiat currency.

With this mindset, one can take the first part of that last hike again and (hopefully) better see what we were driving at. I have no doubt most of you have never looked at Gresham's law from that angle. Please understand that I often converse with people that also present life's trails in just as difficult a format. I hope this helps you.

AdvocatusDiaboli said...

Michael,

"Only unambigious claims on distinct pieces of physical gold are worth anything. There is no gold-denominated credit."

thanks for that first approach. But if somebody wants to either buy or sell gold, there has to be some kind of price discovery aka market place? Because many of the poor suckers will probably want to cash out, to finally live a decent living.

Anyway, you are talking about some future prediction (many folks are already died over waiting for it the last >30yrs), but I am questioning today (or lets say before total collapse):
How it would have to look like today, that it would not "suppress" the todays price in the eyes of Freegolders?

So that FOFOA can be happy today and would complain stuff like "paper suppresses gold".

And if your predictions are right, I wonder why anybody should trust any kind of paper promise for generations at all anyway. If that happens to gold, sh!t I wouldnt let my silver rest in the hands of those crooks that screwed around with gold....
Greets, AD

Jeff said...

You are really acting the fool today, AD, even for you.

for the Giants:

FOFOA: You see, in order for the BIS to supply actual physical gold to each and every giant that was ready to buy, the price would have to rise high enough that someone else with an equally huge amount of gold was willing to become a seller. And right now, at today's prices, we know that the central banks of the world have become net buyers! So the question is, just how high would the price have to rise in order to balance out the demand of the world with the supply, in a physical-only official price discovery market?

for the shrimps and German silver bagholders:

FOFOA: During this time, after the paper market has failed, that GIANT sucking sound you hear when you call your dealer and mention that you have some gold for sale will be the CBs and Giants somewhere at the other end of the dealer network with their unlimited currency, their insatiable demand for gold, and their standing over-bid acting like a giant concubine sucking a golden golf ball down a tiny hose. Let's call these Giants and CBs "the buyers of last resort" for gold. Another said they stand ready to buy any and all physical gold offered for sale.

On the other hand, that sound of thousands of telephones ringing in the background, when you call your dealer to sell your silver, will be all the other shrimps placing their sell orders at the same time. And this is a serious difference between gold and silver. The few giants in the paper silver market are in it for the liquidity and the volatility. It is doubtful that they want to actually hump around 1,000 oz. bars. FOA said that when the paper markets fail, the big money in silver will be selling like crazy to cover the losses in paper gold and scramble into physical gold.

So assuming the paper silver giants are not standing as the buyers of last resort for physical silver at an unknown price, who will be? Will there even be a standing buy order? Or, perhaps, will there be a standing sell order from one giant ETF hoard or another? Will China stand as the silver buyer of last resort, at an unknown price, after nationalizing its own silver mine? Will industry place a standing bid for investment-grade silver after stocking up inventory in preparation for volatility?

You see, the Freegold paradigm doesn't bode well for silver. Paper silver has brought in some pretty big money chasing the volatility trade, driving the price of paper silver way up, imputing that price onto physical investment pieces. But without that pricing (gambling) arena, where will all that big money flee? Will it flee into heavy silver bars (those suckers that the big money buys weigh 68 lbs. each) to support the legion of toy soldiers?

FOA: "…at the very least, the first $10,000 of that figure would represent the current purchasing power of the dollar today. We will most likely get there long before price inflation jumps way up. Once the current dollar gold market fails and gives way to a free physical price, we will see that figure even as our economic function drives all other hard money metals into the toilet. I talking about .50 cent silver. while gold races past its first grand. When we see it we will understand it." (April, 2001 – Gold was at $260)

AdvocatusDiaboli said...

"April, 2001 – Gold was at $260"

jep, and since you love silver bashing, here's one for you:
April, 2001 - silver was at $4.5

still didnt answer the question about appropriate gold price discovery. Want me to repeat the complete question again, or do you want to distract with some more nutty C&P?

DP said...

Guess you didn't get the memo…

Outside parties begin buying gold with dollar reserves,,,,, on the barrel head for 5 day placement. It begins with twenty or thirty 100 ton orders ,,,,,, a billion$$ or so each! Not derivative orders, mind you,,,, hard delivery orders that aggravate and outline the soft nature of modern gold banking. They keep coming,,, days on end! Then, suddenly the paper markets "are no more".

DP said...

Then, suddenly the paper markets "are no more"

Somebody should do something!

Michael H said...

AD,

"Anyway, you are talking about some future prediction (many folks are already died over waiting for it the last >30yrs), but I am questioning today (or lets say before total collapse):
How it would have to look like today, that it would not "suppress" the todays price in the eyes of Freegolders?"


You are being absolutely ridiculous.

You ask how would it have to look like today.

I give you my answer.

You claim that I am making a prediction about the future, and thus my answer is not valid.

Michael H said...

AD, let me reiterate:

Your question is "how would the market look like today".

Which means it doesn't currently look like that.

Thus any answer you receive will be different from what we have today.

Thus all answers you receive can be construed as 'predictions about the future'.

Candy Sange said...

How is that different from Freegold?

Tommy2Tone said...

"Let's call these Giants and CBs "the buyers of last resort" for gold. Another said they stand ready to buy any and all physical gold offered for sale. "

Holy cow, this means AD could make some huge cash profits if he just waits till then to sell.XD

burningfiat said...

AD,

If you really think physical gold is fairly priced along-side paper gold, why don't you exchange some of all your coins for some COMEX futures or some GLD?
With the premise that you seem to purport, that both physical and paper is at fair price at par, the optimal diversification strategy must be to be 50-50 in both paper and physical, no? There is no chance for bigger upside in one or either, right?

Time to stop the smart-ass bragging, and let the action talk! Let's see some of your phys. exchanged for paper!

If you don't diversify into paper, we must conclude you're nothing but a talking Arschloch.

/Burning

Victory said...

...a little clarification in understanding needed

from Greece is the World:

'Rumors have been circulating for a few months now about some large physical buyers on the public LBMA being cashed out with a 25% premium and being sent to the private cash market to get their gold where such a purchase at a premium would not move the official price. This rumor suggests a relative shortage to demand for physical on the official price-setting markets.'

It is my understanding that the LBMA (a private OTC network) was where the majority of physical spot market gold was traded (in addition to paper gold, ie unallocated, forwards, options, ect).

This quote has me second guessing that understanding, so my question is: what is the difference between the private cash market and the LBMA? Also why is this quote using 'public' as a descriptor for LBMA, is that in reference to the publicly posted morning and afternoon london-fix price quote?

I'd like to understanding this process more in general...if the buyer is being cashed out at a 25% premium what exactly does that mean. Is this someone who had an unallocated account who wanted to switch to allocated and then transfer bullion to a vault outside the LBMA member storage network? And where exactly is he being sent...directly to a miner, refiner, or non LBMA private entity?

I realize FOFOA is probably to busy to answer these questions so if anyone else can pitch in it would be much appreciated.

tx

-v

ps. i made reference to a jim rickards interview in a post yesterday in which jim mentioned the BIS being involved in managing the gold price but I forgot to include the link. Sorry about that, here it is for anyone interested:

http://www.youtube.com/watch?v=Y-bjjSL2OUo

Aquilus said...

On AD’s position (time to clean that bathroom again – LOL):
Aside from the annoying habit of asking how things should develop, and why and then when an answer is received slamming the respondent with “but looky, looky here, it’s not like this right now” there are two major things that AD keeps bypassing:

1 Marginal Utility from a Giant’s perspective:

As a coin shop giant, AD cannot seem to let go of the idea that saving all hard stuff is better than saving in gold.

Put yourself in the shoes of a real Giant, with say $100 million coming in every day. So the first day you buy all your cars, yachts, vacation homes.

The second day your airplanes and start onto castles. When done after the first week, then what? Do you bid up commodities like silver and platinum with 100mill a day? How long before you get slapped like the Hunt Brothers?

What else keeps its marginal utility for you? Gold you say? Using today’s market? Do you run the price up with all that cash directly? Or do you just buy “low” and put the rest of the cash in acquiring future gold delivery directly from producers, and spreading your currency purchasing power out in time?

You can also finance gold miners by offering them cash to develop their mines for gold production delivery.

What other choice do you have? You cannot bid up things that impact industry too much, and there are only so many castles and “Scream”s that you can acquire. So either you corner the future gold production or you stop producing. Otherwise, if you become too visible, the current hungry collective may have something to say about your wealth.

2 The fact that the more long contracts are entered in (in the current paper gold market) the lower the price goes .

How can that be? Drop your basic supply/demand baggage for one second and think about it this way. If more traders bid for long contracts at a certain price and each one of these contracts is IMMEDIATELY met by willing sellers (of contracts), does that not indicate to you that there are plenty of sellers at this price?

It sure does, and the price drops as each purchase is met with an immediate and enthusiastic selling (and then some) .

But even the sellers have to keep an eye on the physical market and let that price rise. How? Just curb your selling enthusiasm at a certain price and enthusiastically defend a higher level. And no, these do not need to be naked shorts overall – they can be part of a bigger transaction overall.

And now I give the floor back to AD for the “but look at how stupid the masses are and how we’re getting oppressed more and more”. Oh, and the “hoard all commodities” advice. Come on AD, who said we don’t love you? LOL

Greets
Aquilus

P.S. This was all an off-the-cuff rant, so please excuse grammar and/or run-on sentences.

MnMark said...

Another said they stand ready to buy any and all physical gold offered for sale.

I have a question. If "they" (CBs and giants?) stand ready to buy "any and all physical gold offered for sale", how comes I can still go to APMEX's web site and buy physical gold?

If the giants know that the current price of paper gold is such a massive bargain, why aren't they cleaning out APMEX and the other internet dealers? Sure they may not be able to spend a billion or two there, but why leave maybe millions on the table? Why would the Chinese?

Thanks in advance for explanations of this.

Motley Fool said...

Mn Mark

AD answered that one ( without meaning too I'm sure) earlier.

32k tonnes have been mined in the last 13 years.

Option
A) They take all they can get Now.
b) They take less than they want now with the hope of getting more in future.

Which strategy yields the highest payout?

That is all. :p

TF

Motley Fool said...

Ps. For a long time now the answer has been B) , however declining mining output and greater demand from other sources (ie. John public), means that option A) will soon be a better choice.

Victory said...

MF,

Since giants are not one cohesive homogeneous group that plan together and move in unison Game Theory would suggest the answer to your rhetorical A or B question as posed is not so rhetorical, in other word - A

...therefore there must be more to it than that

-v

AdvocatusDiaboli said...

A,
"What else keeps its marginal utility for you?"

another 400oz bar? Comme on, seriously is that's how you look at the world?
I guess it depends on your very special personal tick. After a certain point you always reach a certain marginal utility and start to wonder, what is that good for. Some people reach that earlier, some people later and some poor lost souls never. But to think that there is any "giants" with 1000t (or whatever), that will buy the next 100t at a price twenty times from today (in terms of purchasing power), you dont need to be a genius to figure out, that that will never ever happen, no matter what some nutty guru might have told you on his trail.

And about BF's comment, why I dont hold paper gold? I dont see much sense in it? The idea of any PM is to avoid counterparty risk. Am I the first to have figured that out? According to BF, I must be the first and one&only genius, since all those dumb giants just hold the paper and will storm into the BB.
BTW: funny in Germany that due to the completely f*cked up tax laws for paper gold you have to pay capital gain taxes, but for physical gold you dont...and: next for europe will be a totalitarian socialist €regime or a break up of the euro, so better not holding too much paperish at all anyway.
Greets, AD

Motley Fool said...

Victory

Certainly not a homogeneous group. However as soon as the answer of gold presents itself, a few minutes of thinking shows that option B) is indeed better...for now.

So, the only relevant giants are those wanting to procure gold.

Now consider. Say you are correct, and A) is best Now. What would happen if PBOC offered to swap all their USt holdings for gold with immediate effect? How much gold would they get? ( Sure I know, they are trying to do it on the sly...but there is limits to what they can do.) 10 tonnes? 100 tonnes? I doubt that much would be delivered before the realization sets in of the true value of gold and it is bid to the moon, and concurrent sell offers are withdrawn.

Doing it on the sly...they have imported more than 300 tonnes this year already( just from Hong Kong- they don't publish their trade statistics on gold).

I'd say option B) is working nicely...for now... yet at this pace that for now won't last much longer.

TF

Aquilus said...

AD,

With vision like this, I think you should keep impressing the newbies with how many 400oz bars you bought last week at the corner coin shop, LOL :
"After a certain point you always reach a certain marginal utility and start to wonder, what is that good for."

Ok, but seriously now, let me explain it to you in "wealth for dummies": old money, wealthy families still have the coins that their grand-grand-grand-parents bought hundreds of years ago still. The SAME coins! Because they earned more than they can possibly spend and decided to store the excess into the only medium that

a) keeps its marginal utility at their scale
b) can carry the purchasing power across decades and centuries.

That's what marginal utility does for you. Not iron bars, not bushels of corn, not livestock.

So, no, you cannot see that, because you have to push all that baggage out of the way, and it's too heavy to move...

---------
Now BF is very capable of answering for himself, but I want to point out that
a) here you go with the current oppression again
b) your "socialist" prediction assumes that producers will continue to want to produce and have excesses inside Germany because ???? Social responsibility? Gun to their head? AD asked them to do so nicely? Sorry to say but you are sooooo naive AD.

If people could be forced to be REALLY productive and innovative under duress, North Korea for example (that fits your glorious description of the future ) should be ruling the world by now. Come on, come up with something better than this.

Greets, future comrade AD

burningfiat said...

AD,

BTW: funny in Germany that due to the completely f*cked up tax laws for paper gold you have to pay capital gain taxes, but for physical gold you dont...and: next for europe will be a totalitarian socialist €regime or a break up of the euro, so better not holding too much paperish at all anyway.

Aha! Got you!
Let's list all the scenario's:
1) FO(FO(A)) is right #FGFTW -> hold phys. gold
2) Status quo (slowly rising $PoG) -> hold phys. gold
3) AD (mean preussian FOFOA critic) is right:
3a) Nasty socialist hobbits stealing precious -> hold phys. gold
3b) €-Shire breaking up -> Chaos -> hold phys. gold

WTF are we arguing about? You're not even willing to hold any paper (not even 1%) yourself!
So, you admitted that you agree that phys. has more upside potential than paper? Greets!

According to BF, I must be the first and one&only genius, since all those dumb giants just hold the paper and will storm into the BB.

No, on the contrary. You're surrounded by people who made the same choice as you right here at FOFOA's -> Go physical!
I think there's a reason that you ('re subconscious mind) chose this place to hang out :-)

enough said...

VTC and/or VICTORY,

Any additional physical outflow from GLD today?

Where can I find the data? I've been relying on the lovable but increasingly unreliable Mr. Organ......

Victory said...

enough,

2.5 more tonnes gone today

http://www.spdrgoldshares.com/sites/us/value/historical_archive/

table updated everyday after market close.

MF,

I'm not saying A) is the answer today obviously it's not or GLD would be empty I'm just saying Nash Equilibrium would suggest there is more to it than the A or B dichotomy you presented. Unfortunately I'm not smart enough to know why:(

From DP's quote of FOA above:

'You see, all it takes is for one or two government and/or private entities to pull the cord. Most all of you long ago came to the same conclusion; a Dollar / Euro currency dispute could set this off. Outside parties begin buying gold with dollar reserves,,,,, on the barrel head for 5 day placement. It begins with twenty or thirty 100 ton orders ,,,,,, a billion$$ or so each! Not derivative orders, mind you,,,, hard delivery orders that aggravate and outline the soft nature of modern gold banking. They keep coming,,, days on end! Then, suddenly the paper markets "are no more".'

So what will be the spark that starts the run? I wonder if it will be plain old American capitalist greed...when do the hedgies see what FOFOA sees? Why has this not happened yet - well my simple mind falls back to a 'simple is as simple does' explanation...Occam's Razor....they just haven't figured out what FOFOA sees?

-v

costata said...

DP,

Thanks for bringing the issues behind my questions into sharper focus. I'm working on something. So I may be tossing a few things like this into the discussions in coming weeks. Apologies to all in advance if they are off topic.

BF,

You were way ahead of me on this but just to drive home the point:

BTW: funny in Germany that due to the completely f*cked up tax laws for paper gold you have to pay capital gain taxes, but for physical gold you dont...

Congratulations to AD you are doing exactly what the Euro Freegold-RPG architects want you to do. You are getting out of their way while they conduct their currency war(s).

I also want to give a big SHOUT about this report:

Kitco News) - While this has not been widely reported in the Western media, news broke this week of a massive illegal gold-futures trading scam in China. Not only does it underscore the growing hunger for gold among the newly minted Chinese middle class, but also hits home the rationale for owning physical gold, according to one U.S. based asset manager.

Over 5,000 investors were bilked out of 380 billion yuan, or $59.62 billion in a scheme involving Loco London gold since 2008, according to a report in the China Daily.


http://www.kitco.com/reports/KitcoNews20120720KB_china_gold_scam.html

Does anyone have any further information on this?

costata said...

The scandals just keep piling up, don't they?

costata said...

AD,

One more thing. Uncle costata bought gold for the first time and continued buying over a period of time for precisely the same reason you gave for holding it - no counterparty risk. Before coming across A/FOA and FOFOA's writings. One reason only. No counterparty risk!

So here's a little homework excercise for you. Name the other asset reserves with no counterparty risks. I'm determined to see you blossom into a star student here at the FOFOA School of the Aether.

http://en.wikipedia.org/wiki/Aether_%28classical_element%29

costata said...

There's an old saying in Australia: You need an old dog for a hard road.

http://www.jsmineset.com/2012/07/20/investors-springfreedom/

That saying comes from Ireland apparently according to a quick Google search. This Irish proverb:

The old dog for the hard road and leave the pup on the path.

http://islandireland.com/Pages/folk/sets/proverb.html

Watch this space that Jim Sinclair is creating would be my advice to anyone inclined to write off "Mr. Gold".

ampmfix said...

Friends, allow me a little off topic taste, did you know Ralph Towner?, I have loved his music for almost 40 years, please enjoy!

http://www.youtube.com/watch?v=uBGWkxwL7vU&feature=relmfu

http://www.youtube.com/watch?v=5DE7ScoLmp8&feature=related

Best regards.

ampmfix said...

oops, this might have been more appropriate, green and golden (the trail and the quest of course)

http://www.youtube.com/watch?v=Hegtu7bSLZE&feature=channel&list=UL

Signing off...

Aaron said...

ampmfix-

most excellent link. i had never heard of ralph tower before today but i really like his style.

have you heard of michael hedges? i had the pleasure of seeing him live in manchester in the early 90s before his untimely death.

http://www.youtube.com/watch?v=YaIN13aDbCc

http://www.youtube.com/watch?v=jN3439l4HR0

costata said...

More on that gold trading scam in China:

http://www.chinadaily.com.cn/china/2012-07/18/content_15592501.htm

Police in Central China's Henan province rounded up 33 people suspected of illegal gold-futures trading in a case involving more than 5,000 investors and at least 380 billion yuan ($59.62 billion).

The suspects, who had been trading since October 2008, had never registered with industrial and commercial authorities, as required by the law, said Guo Congbin, director of Luoyang public security bureau, on Tuesday.

They promoted investments in Loco London gold, the London gold market in Hong Kong, Guo said, which is illegal on the mainland.

According to Guo, the suspects claimed to be agents of overseas companies dealing in London gold and, with promises of huge returns, persuaded investors to establish accounts on the Internet for trading Loco London gold.


h/t JSmineset for the link

costata said...

This may only be an incremental step in loosening up controls over the banking system in China but IMO it shows that progress is being made.

http://www.ft.com/intl/cms/s/0/9d707c16-d191-11e1-bb82-00144feabdc0.html#axzz21EzjhBhu

At present banks are not allowed to trade bullion, or any precious metals, directly with each other and can only buy gold domestically through the Shanghai Gold Exchange or the Shanghai Futures Exchange.

However, about three weeks ago the exchange and other financial institutions circulated a draft of new regulations for interbank gold trading to solicit comments from commercial banks, according to a metals trader at a large Chinese bank.


h/t JSmineset for this link too

costata said...

This interview is mindblowing. Forget the first 12.45 minutes. It's the usual Max and Tracy stuff. The interview with Jim Rickards is stunning. A must listen IMHO.

He discusses gold in relation to Tier 1, the EU versus the rest and LIBOR. In the LIBOR discussion he explains, from a legal perspective, why it is highly unlikely that the banks will be able to quash litigation against them.

http://maxkeiser.com/2012/07/19/kr316-keiser-report-alien-bankers-leave-earth-alone/

BTW if someone provided a link to this earlier my apologies for missing your heads up.

AdvocatusDiaboli said...

costata,
"Congratulations to AD you are doing exactly what the Euro Freegold-RPG architects want you to do. You are getting out of their way while they conduct their currency war(s)."

See, stuff like that always confirms to me, that FG folks might just be some lunatics and see stuff in things that they are absolutely not there!
I threw that fact in, just to see the people who have no clue about the real life (in europe) and issues jump it.
As a small hint, I even especially pointed out: "fucked up tax laws", but still you serviced your cult regardless of the background knowledge or maybe to care about back questioning it.
That background on this taxation issue has absolutely nothing to do with gold at all and applies to many other things as well (e.g. any other commodity, houses.....) BTW: Right now it is being under discussion by the high financial court on how to deal with physical backed ETFs that allow delivery.
Somebody want some more of that bait? How about that, for the "silver-is-money-bugs": silver bullion coins are heavily VAT reduced in Germany....

And about your approach: "Name the other asset reserves with no counterparty risks."

How about that: It does not exist and even gold has "risk", it is just about minimizing towards the risks, you want to hedge and what your personal believe system of value is.
Greets, AD

AdvocatusDiaboli said...

BF,
"WTF are we arguing about?"
I (only) arguing about the topic of the FOFOA post "does the current paper market supress gold?". I am not arguing about assumptions if price of gold will rise or fall. I am also not arguing if owning paper is better or equal or worse than physical.
Greets, AD

Motley Fool said...

AD

Just to be clear, what is your actual position? Is physical gold better, is paper gold better, or are they the same?

I'm sure I'm not the only one that is confused as hell as to what you are actually arguing about.

TF

AdvocatusDiaboli said...

MF,
can you read? I wrote: I am not arguing what is better. That is absolutely not my point. But amazingly(?) the FG folks want to distract from my point: "is paper suppressing the physical? I do have serious doubts about that assumption".

But since you ask: It depends for everybody personally. Paper gold is more liquid in todays real life (just a click and in any smaller size tradable), but has a counterparty risk. Physical gold is less liquid, but does not have the paper golds risk, but instead the reallife risk of being stolen or subject to capital controls or similair oppressions in your personal jurisdiction (but that might apply to paper gold as well, if it is tight to your jurisdiction).
For me personally physical gold is the only way to go, <= as I said, for me!

What gives me more a headache is to decide for me the question about paper silver.... so far I also exclusively hold it physical because it is great for stacking. I guess, if I want to engage in copper I have to stay the paper way...
Greets, AD

Nickelsaver said...

I believe AD's point is that no one can know, which would make him an agnostic. But he sure has a lot to say for someone that believes in nothing.

Jeff said...

Rickards sounds FOFOAish, but of course he throws in SDRs and the dollar to keep from baldly stating the truth.

IMO he is wrong when he says China is a weak hand. China has the most gold, after the US and Europe. China is spending dollars to buy resources, gold, GDP growth, and time to buy more of all of these. They are no longer structurally supporting the USD and are in runoff mode. They will stay in runoff mode until the system crashes. Who knows how long that is?

Motley Fool said...

AD

Yes, I know, you keep covering old ground due to your lack of understanding.

From my perspective it does seem like willful ignorance given all the people trying to help you.

It's not an assumption, it's a simple bit of logic.

Paper gold can be created at will, gold must be extracted, so the 'production cost' for these two dissimilar items differ vastly. The current system would have you believe the paper claims are fungible, and that is true, as long as only a small percentage of claims is made.

For those that want to use gold to preserve their wealth, not those that only want to speculate and simply use these as instruments of price speculation, paper gold does them the disservice of their belief and comfort that they own the underlying asset.

The reality is that all claims ( of those that want gold..not the traders) cannot be settled. Hence these paper gold instruments are satisfying some of the demand of those that actually want physical gold.

Since the 'production costs' differ this means that this paper gold structurally undervalues the underlying physical gold.

It's a simple bit of logic. All one needs to do is Think about it, something you seem unable or unwilling to do.

TF

Motley Fool said...

Ps. A relevant FOFOA post, talking about paper chickens. :P http://fofoa.blogspot.com/2009/08/call-of-century.html

AdvocatusDiaboli said...

MF,
only looking at what you wrote, applies to paper orange juice or paper coffee just as well. And yes, I know the difference of those commodities compared to the useless commodity of yellow stones, but you did not touch that issue in your explanation anyway.

So your explanation on what you wrote is actually really worth ....NOTHING!

That's okay, that your explanation sucks, but please dont call others therefore "lack of understanding" or "ignorant".

Greets, AD

Motley Fool said...

AD

If you continue to skim my words without giving it any Thought, you never will reach understanding.

For both gold and oranges there are some who use paper instruments to speculate on the price. I said we can ignore this similarity.

The second group in both markets is those who use the paper instruments to take delivery of gold or oranges. These are people who want the underlying physical good. This similarity too we can ignore for the purpose of this discussion.

There is however a third group which we cannot ignore in gold space who has no counterpart in orange space.

This is the group who hold paper promises thinking they have hold of the actual physical underlying thing.

In the orange market there are no people who hold contracts on a similar mindset. They either want actual oranges or they want to speculate on the price.

This third group in the gold space is reducing pressure on the price of physical gold by accepting paper tokens in lieu of it, thinking they are the same. Which I hope we agree they are not.

By this means, structurally, the price of physical gold is undervalued.

TF

Motley Fool said...

Ps. Technically we cannot ignore the first group either, but as you say this effect is similar in both markets.

This means it's okay to ignore when comparing oranges vs. gold, but not okay to ignore when considering oranges versus say haircuts.

AdvocatusDiaboli said...

MF,
that "third" group might just be in your head?

Because I am confinced, that this "third" group is basically just the same as the first group: Buying paper with paper to get paper for paper.
Greets, AD

Motley Fool said...

AD

No.

Unallocated gold holding is an example of the third group.

There are no unallocated orange contract holders.

TF

AdvocatusDiaboli said...

MF,
maybe, maybe not. I dont know how "unallocated gold" is "mined" and who buys or sells it for whatever reasons and how those identities account it. But for somebody from the keyboard commando and to claim to know it all, pretty thin claim of you.
As I said way earlier: Wake me up for FG, whenever the IMF decides to change the accounting rules for gold vs. receivables.
Greets, AD

Motley Fool said...

AD

Did you bother to read the link I posted?

Can you see any reason why some 'wealthy' would hold such contracts thinking it was actual gold?

Could you at least suspend disbelief and consider the implications of what I am saying, if I am right?

Can you see why paper gold Lowers the price of gold if the above is true?

Your flaky unappreciative response to my well reasoned explanation for this issue you have been harping on for days is curious.

At the least we have discovered where our premises differ. Now proof either way would show whose view is more correct.

TF

Motley Fool said...

Ps. Your response amounted too : I don't know (and can't be bothered to find out)...so let's do a ad hominem attack, and dismiss this issue as irrelevant (since I may be wrong) and move on to another of my pet peeves (that today is not tomorrow).

You were saying something about weak? Haha!

AdvocatusDiaboli said...

MF,

"Can you see any reason why some 'wealthy' would hold such contracts thinking it was actual gold?"

normally wealthy people are suspicious. I have no idea why in this case they shouldnt. I am, but I dont even hold 100t of gold.

"Could you at least suspend disbelief and consider the implications of what I am saying, if I am right?"

Sure, basically your idea is just as good as the Crash-JPM-silver stuff.

"Can you see why paper gold Lowers the price of gold if the above is true?"

Only in case I assume massive "fraud" like you do. But in that case it is the fraud that suppresses the price, not the "paper principle" in itself.
Greets, AD

Motley Fool said...

AD

I don't know why you paint it in negative moral terms.

Banks operate on the idea of a profit-motive and that informs their actions.

For a very long time now only a small percentage of paper contract holders have actually stood their claim. Banks have become very adept at guessing what percentage of contracts will be redeemed (and not expire worthless). Simple profit motive.

The truth is that for a long time now this system has worked. No one was defrauded. Everyone got what they wanted. Some got physical, some got to speculate on price, and some kept on option on physical.

What they did not take into account is that some of those holders may not take delivery now, but will want to do so at some point in the future. They have not held gold on reserve for those, because they have seen no evidence of their existence.

This is now changing as the realization of counter party risk is coming into view (with debacles such as MF Global and PFG).

As you pointed out there is the matter of theft as a risk of physical. Furthermore there has been confidence in the system ( there still is for the most part). Furthermore transaction costs for taking delivery physical have been higher.

These are three reasons why some wealthy would have held these contracts rather than psychical.

As I said, recent market events have made some revisit those assumptions (look at demand of ETF's vs physical over the last few years for example, and how that has changed).

TF

burningfiat said...

AD,

What would kind of change in IMF accounting rules for gold vs. gold receivables do you think is necessary for FG to come in to play?

In my naive opinion, all the necessary rules are in place for Freegold to take place. Euro established, No VAT on gold, MTM gold at ECB and lots of other CB's worldwide. The weather is fair and walking in the footsteps of giants is easy, don't you agree?

But please enlighten me. What are the Freegold-showstopping IMF rules?

Greets!

AdvocatusDiaboli said...

MF,
I completely agree with you, if all your assumptions are correct. But they are just "assumptions"!!! from somebody from the keyboard commando.
You assume that the unallocated gold is not ready for delivery or does not even exist.
You assume that people buy it, because they want the physical (for whatever reason, also very questionable).
Therefore you assume that therefore all paper will burn....

Okay I hear you, and as I said, maybe, maybe not. You assume something have no prove, but state it as a claim, pretty bad attitude.

But anyway, that has ABSOLUTELY NOTHING to do with the general claim "paper is suppressing the gold market".
Greets, AD

Motley Fool said...

AD

You are correct, it is not absolutely provable, and will only be possible in retrospect.

However there are a lot of anecdotal evidence for my claims, and none that contradict it(that I know of).

Things such as the shift in physical versus paper demand, the amount of contracts versus stocks ( as far as we can know it), etc.

I just explained to you the mechanism whereby that part of the paper market supresses the physical price, and linked you to a post by FOFOA that explained how the first category also does (which you agreed too, just pointing out that it is also valid for orange juice).

Your claim that it is irrelevant is nonsensical.

TF

Motley Fool said...

AD

If you dispute the theory it is up to you to provide evidence that contradicts it.

We will all welcome evidence to the contrary if you are able to provide any.

Until such time the theory is valid.

That is how science works.

You can't say it's incorrect because it is only a theory, that means you misunderstand what the concept means in science.

If that is your position then you can go jump off a building since the theory of gravity is only a theory.
I'm sure your belief that it is only a theory will alter the result, not.

TF

Aquilus said...

AD,

Motley is right in challenging you to back up you claims.

For example: "You assume that the unallocated gold is not ready for delivery or does not even exist.
You assume that people buy it, because they want the physical"


Many time in this blog unallocated gold has been explained (along with how BBs can sell unallocated gold and not be naked short). Also recently, there was the admission of LBMA of demand for over 7000T of gold in a short period. There have been explanations of the mechanisms of how all this works.

Now it's your turn to explain your unproven allegation, or if not, then admit that you cannot.

What's good for the goose is good for the gander.

I'm all ears, and I will call you out if you try to change the subject, obfuscate, build straw-men, resort to attacks on others and all the other avenues you like to take to avoid the subject.

The floor is yours: show us how unallocated gold is ready for delivery. In case you have a short memory, here is your quote again: "You assume that the unallocated gold is not ready for delivery or does not even exist.

Anonymous said...

Herro AD! :D

This was a choice quote of yours:

But to think that there is any "giants" with 1000t (or whatever), that will buy the next 100t at a price twenty times from today (in terms of purchasing power), you dont need to be a genius to figure out, that that will never ever happen, no matter what some nutty guru might have told you on his trail.

Once again you reveal how little you comprehend. You think that the reason A/FOA/FOFOA give for the future high price of gold is that giants are going to be "bidding it up". While they will still be buyers, the high price will be more because they are simply not selling i.e. they are strong hands.

Further, whomever buys that 100t will perform only one calculation: will it preserve purchasing power? Weight does not matter, only preservation of purchasing power. So if I net-produce $50,000 in one year and I want to save it, as a saver it does not matter whether I can get 1 oz. or 50 oz. for that $50k so long as it preserves its purchasing power.

ampmfix said...

Hi Aaron,

Thanks, very interesting, I heard of him but never listened carefully. Another heterodox musician with a notably interesting instrument.

Cheers.

Anonymous said...

****TROLL ALERT****TROLL ALERT****

AD is a troll, and as such he is here to do what trolls do. His mission is to antagonize, disrupt, and otherwise draw attention away from the subject matter of this blog in order to refocus it upon himself.

He is not the first troll to have taken up residency at this blog, nor will he be the last. He will eventually go the way of all the other trolls when he no longer gains sufficient satisfaction from the amount of attention given to him. The less attention he receives the sooner he will leave.

****PLEASE DO NOT FEED THE TROLL****

Edwardo said...

Here's the final paragraph from Bruce Krasting's latest entry in which he assesses what form and quantity of monetary largesse is next from The Fed.

"The entire global financial system is based on fiat money and the presumption that the money has “value” as a store of wealth. Nearly every action by the Fed over the past few years has led to the debasement of money. In the final stage, the issuers of money debase it to the point where it is no longer desirable to hold. I see the move to negative rates across the globe as a tipping point, one that will be damn hard to reverse once undertaken."

http://www.brucekrasting.blogspot.com/2012/07/bernanke-post-schumer-gaff.html#comment-form

AdvocatusDiaboli said...

MF (&others)
you are correct that I cant prove anything, neither can you. You tend to interpretate certain observations as indication for your theory. That's basically all. Again, the original post "one is not like the other - paper suppresses" is similair: It takes a judgement/assumption on the motives of the holders of above ground stacked and certain motives of the buyer and sellers of "physical promises". Can those be considered a "landfill" or "savers" or "speculators" or "religious tokens fanatics"? Let's face it: Nobody can tell for sure. I can tell you for sure about my personal actions: Mine is not for sale for holding fiat at any price (especially that crappy €) and I will not buy at multiples of production costs no matter how much money I have left to spend/save/invest PERIOD. So in todays markets you can consider me a "landfill". For other holders freegold has already arrived, because there is no other explanation for the "old-coin-phenomenum".
One of your next approaches is to interpretate what is happening as that all changing "naked short squezze". Maybe, maybe not. Let's look at the silver crowd, boy they must be pissed after silver fell 50% and you can get as much physical as you want. Although we propably agree that this action in this short period of time this supply has nothing to do with a "demand driven commodity in a paper market".
Having this in mind your FG inverted waterfall is also more than questionable. But do I care if gold goes to 50% and lots of physical available? No. Can you say the same about yourself honestly? That's the question to pose to yourself and than compare it with your own motives on what to believe.
Oh and about another comparision "paper suppresses": Isnt it amusing that so many jerks around the world shout, that "paper food" is supposingly raising the food prices for the third world? But for the "GATA folks" it's the opposite? Looks like everybody sees what he wants to see, just look at my latest response to costata.
Greets, AD

AdvocatusDiaboli said...

matrixsentry,
you call me a troll, because being the george carlin at a christianity blog?
So who's the troll or should I better say who's the retard?
Greets, AD

AdvocatusDiaboli said...

MF,
and to add to what's going on in the market: I guess FOFOA sees it basically just like me, because to close that flaw about the mining supply to the "landfill" (paper) market, he just makes up this crazy assumption of: All gold mines will be world wide nationalized/taxed to infinity.
Guess why he needs this assumption for his FG theory?
Greets, AD

Victory said...

JR,

I remember a long time ago someone asked you how you were able to search through the archives so precisely and you replied with instructions (i believe it involved google?)

Of course if I new how to search for that reply of yours I wouldn't need to find it in the first place.

If I am remembering this correctly and not making it up would you be so kind as to direct me to it.

tx,

-v

Anonymous said...

Engaging AD in an effort to illuminate the points we explore on this blog is a complete waste of time. However calling him out can be constructive in that recent visitors to our blog can more easily see what is crystal clear to those of us that have endured him since his arrival.

I do not mean to say that all response to this miscreant is tantamount to feeding a troll. Specifically, it is engagement that seeks to sway him or to solicit a cogent argument from him that leads to a more fulfilling exploration of our chosen focus that is counterproductive. I encourage everyone here, veteran posters as well as new arrivals, to blast away at his manifold failings both as a seeker of knowledge and as a human being.

Perhaps he has redeeming value in the sense that he plays the Devil's Advocate. But just try to convince the Devil that he should be something other than the Devil. Those with patience (I am not one) can dissect this curious creature to see what makes him so different. I will watch with amusement and perhaps morbid curiosity. However I will be simply bored with attempts to "educate", illuminate him, or otherwise appeal to his sensibilities in an effort to change him into something that he cannot be.

There is a lot to learn and too little time I fear to coddle a fool.

Victory said...

Greets, AD

..heheeeeee!

AdvocatusDiaboli said...

matrix,
congratulations:
"leads to a more fulfilling exploration of our chosen focus that is counterproductive"
...nothing to add, Sir.

seriuously, goes none of the "followers" feel kind of stupid by now reading stuff like that?

costata said...

matrixsentry,

I hear you. I'm over AD as of now. His responses to (for example) MF were puerile.

It's a shame in a way. You can start this journey with only counterparty risk in mind and follow that one "trail" all the way to the central bank/currency issuer (CB/CI) and arrive at this question:

What will a CB/CI do when it sees counterparty risk appearing in its books?

You can validate the predicted action by inductive reasoning. Simply compare gold to the other hard assets with no counterparty risk. By a process of elimination you arrive at physical gold as the solution to the counterparty problem. You can then further validate the conclusion by looking at network effects and other considerations.

Theoretically SDR could be an alternative. But that "solution" has to survive the gravitational field of the too hard basket. It requires agreement among a large number of parties to be viable. And it may only require one key party to reject it in order to make it unviable for all parties.

Once you get to the CB/CI at the top of the trail and see that gold is their only option you find the liquidity issue resolved permanently. The CB/CI is the only customer who can never run out of "money" having no objective limit on how much money they can create to purchase gold.

Like I said, it's a shame but there's only so much you can do.

ampmfix said...
This comment has been removed by the author.
Anonymous said...

Some current affairs. There is a rumour that the IMF will no longer back the ongoing support for the Greek government. If true, this means that either the Euro zone governments will have to pay more or that there will be further writedowns. In this case, the ECB itself might be affected as they presently hold some E90bn of Greek government debt (number from memory, hope it is correct).

Victor

Motley Fool said...

AD

Are you a woman? :P ( no offense intended ladies)

I cannot comprehend how one so devoid of basic reasoning capability can claim to be a small industrial giant.

Perhaps the answer lies in inheritance.

I'm joining the growing consensus, and will try to stop myself from engaging your puerile bullshit in future.

TF

Edwardo said...

VTC,

Who or what is the source of this rumor?

AdvocatusDiaboli said...

Singapure appears to set PM VATfree:
http://www.iras.gov.sg/irasHome/page04.aspx?id=2002#budget_2012
lucky singapures, additionally to gold they can buy also silver&platinum VAT free, different from europeans :(

Clyde Frog said...

Interesting. How about their CGT treatment?

Anonymous said...

Long time reader and twitter follower, first time poster, still a week or more behind on reading comments, so I've likely missed much.

costata,

I like the exercise you presented regarding interest rates and what's left after the principle is settled. It's a topic I've been pondering for years. I think the answer is, "demand".

JR and FOFOA have been right to point out that no new type of money has been created or left behind as a result of the interest. What has been created though, is a future demand for more money (credit or base). It presents a legal obligation on the part of the borrower, and likely on the creditor as well who will have pledged that income as collateral for their own debts.

I think that this demand on future money is critical for understanding where we are today and will be tomorrow. For if there is ever a failure of credit money to deliver the volume needed to satisfy the demand for said obligations, then the market price (i.e. interest rate) will come under pressure to attempt to increase supply.

Put another way, we know that Gold bids for dollars and can selectively go in to hiding if the price is not high enough. The interest dynamic is exactly the opposite though, as the built up burden to satisfy obligations causes there to be a bid for dollars *at any price*. In effect, it's worth driving the current interest rates to near zero in order to try to create enough new credit money to satisfy past obligations. And when new credit also goes in to hiding because the price is too low, base money is printed, because the collateral chain of obligations threatens the very legal system on which government legitimacy is predicated.

So to answer your question, it's not a new money, but a demand for new money, a demand which creates and responds to the exact opposite dynamic as gold.

I haven't followed this through to negative nominal interest rates, as every time I try to push my mind into that dynamic, I feel like I'm crossing the event horizon.

costata said...

Harpua,

Thank you.

enough said...

Just one comment on AD.....

He must be encarcerated and in 23 hour lock down.

Who else would have the time to write long diatribes 23/7/12

If he is not encarcerated then I suggest that AD goes out and enjoys the journey that is life and makes an effort to avoid carpal-tunnel.

I'm going for a swim with my labs now. Everyone, get out and enjoy the day......best, E

Victory said...

A while back Aristotle asked this forum to give some thought as to why China was given direct bidder status by the Fed. I'm not sure if that question was ever answered sufficiently (apologies if it was) but I read this article yesterday and was immediately reminded of that thread:

http://azizonomics.com/2012/07/21/why-is-the-fed-not-printing-like-crazy/

It seams clear to me now that China being given direct bidder status is like a high-roller getting comped a suite at a Vegas casino - just keep playing baby!

The article also linked to the wiki-leaks US embassy cable in which former Ambassador to China and presidential candidate John Huntsman relayed the end China's structural support for US Treasuries. The cable was rather blunt and struck a cord with me.

props to Turd for providing the link!

here is a snippet from the wiki-leaked cable:

(A crucial move for the U.S. is to shift its crisis to other countries – by coercing China to buy U.S. treasury bonds with foreign exchange reserves and doing everything possible to prevent China’s foreign reserve from buying gold.

If we [China] use all of our foreign exchange reserves to buy U.S. Treasury bonds, then when someday the U.S. Federal Reserve suddenly announces that the original ten old U.S. dollars are now worth only one new U.S. dollar, and the new U.S. dollar is pegged to the gold – we will be dumbfounded.

Today when the United States is determined to beggar thy neighbor, shifting its crisis to China, the Chinese must be very clear what the key to victory is. It is by no means to use new foreign exchange reserves to buy U.S. Treasury bonds. The issues of Taiwan, Tibet, Xinjiang, trade and so on are all false tricks, while forcing China to buy U.S. bonds is the U.S.’s real intention.”)

The article also brought into focus for me the 'print' or 'not to print' rock and a hard place Bernanke is in.

from the article:

'And that, in a nutshell, is why Bernanke is not printing nearly as much as Krugman wishes. In my view only a brutal 2008-style collapse can bring on the kind of printing — QE3, NGDP targeting and beyond — that the pro-Fed blogosphere wishes to see, because it is only under those circumstances that China and other creditors will happily support it.'

So the situation appears to be as such:

1. No domestic and especially no international political will for more Fed QE unless there is a crisis
2. The USG needs someone to buy it's bonds because they have a trillion-dollar funding gap, if it's not the Fed or China who's it going to be?
3. Will the Fed cut the interest rate paid on deposits at the Fed to zero to steer US domestic banks into funding the USG debt addiction - as obnoxious Michael Pento is so certain of?
4. Will the Fed say eff' the political will and be proactive and buy UST anyway or will they force a market crisis situation first as a backdrop for cover?


I realize this is nohting new to the readers here but I'm curious to know what others opinion are. We have a fed meeting coming up Aug 1st and the Jackson hole meeting towards the end of Aug

-v

JR said...

Hi Victory,

Google search with a few key words or phrases and either:

blogurl:http://fofoa.blogspot.com
site:fofoa.blogspot.com

If you know the a couple words, you cans search for the text in quotes.

J.R.

JR said...

Some dogs are smarter and more eloquent than others - thanks!

There's a dog in the station!

Jimmy...Jimmy
Yes, Dad
Jimmy, I have some bad news...
What might that be...Dad?
It's about your cat, Poster...
You wouldn't be talking about Poster Nutbag, now would you?

Your cat died!
Poster is dead
Poster is dead
Poster's so dead
How about a goldfish?

I don't want a goldfish
How about a goldfish?
I don't...want a goldfish
How about a goldfish?
I don't want...a goldfish
What do you...
What do you...
What do you...
I want...
What do you...

A dog
A dog
There's a dog in the station

Victory said...

Thanks J.R., do you now of a way to include the comments section in the search results?

JR said...

AFAIK it only searches the post and first 200 comments, not sure how to get it to search the extra comment pages after comment #200.

db said...

Hi all, even though I've gone trough all posts and comments and revisit FOFOA's everyday to keep up with the always enlighting discussion, there's still something that keeps coming back to my mind.

Not that I don't get what FG is about, or what the final outcome will be, but the question that Victory raised  a couple of days ago to which nobody has made a single comment it's that something I keep thinking about ....

Maybe her comment was considered naive or you've considered it's not worth answering it, but given it's  something I've also asked myself I thought we 2 might not be the only ones.

"So what will be the spark that starts the run? I wonder if it will be plain old American capitalist greed...when do the hedgies see what FOFOA sees? Why has this not happened yet - well my simple mind falls back to a 'simple is as simple does' explanation...Occam's Razor....they just haven't figured out what FOFOA sees?"


To share my view ( if anybody cares) I don't think it's because they haven't figured it out ... Know some of those HF guys and they're usually smart people, so must be one of these:

a) Some of them are aware but since clients demand nominal performance, they've made the move with their own personal savings.

b) Same as a) + some of HF's proceeds into phys

c) Same as b) but as Berkshire's being too big they'd be the giant running out of the BB screaming: "There's no gold"

d) They see FG in such a distant future that they're not pondering it for the time being.

e) They see a completely different outcome

JR said...

Good thoughts db,

FOFOA explored the distinction between investing and saving, using Warren Buffet as a disucssion point Yo Warren B, you are so OG!. He explored an idea introduced in Glimpsing the Hereafter and The Studebaker Effect:

"A saver is different from an investor or a trader/speculator. A saver is one who earns his capital doing whatever it is he does, and then aims to preserve that purchasing power until he needs it later. Investors and traders aim to earn more capital by putting their already-earned capital at risk in one way or another. This takes a certain amount of specialization and focus."

==========================

One related idea that sorta also gets at the "why not" from another angle is from GLD Talk Continued:


A/FOA said the ECB/BIS strategy was to “expand and support” the dollar paper gold market so the dollar would eventually “bankrupt itself” just to keep the gold market going and stay in the game with the euro.

FOA (08/13/01; 07:24:30MT - usagold.com msg#96)

A very large part of that war strategy, employed by the ECB/BIS, was to let the dollar / IMF faction hang themselves by expanding and supporting the whole arena of this dollar paper gold market [the ECB/BIS is supporting and expanding paper gold as a strategy]. Inflating the gold market place with so much "paper gold" that we would eventually have to bankrupt ourselves just to keep the dollar in the war game against the Euro.

[…]

So, don't count on this destruction of our paper gold market to mark the real value and availability of physical gold; that ratio will split somewhere down the goldtrail. This action will scare most harden gold investors to death; especially the ones in leveraged gold stocks and lesser white metals!

The war between gold and the dollar has been over for a while now. The action, today, is between the dollar and the euro arena and this is what will break the price lock on gold. Leaving gold bugs with a lot of questions that ask why this: both systems will strive for a higher currency price for gold; one doing it because they have to; the other doing it because they want to! The casualty on this battlefield will be the world gold market as we know it. A market caught between how Western perception thinks gold's price should be "discovered" and at what price level trading in physical gold craters the entire paper structure. A structure of American based "paper gold".

We have been saying for some time that this will be "the" show to watch unfold; but only if your holdings allow you to stay still in your seat as it happens (smile).

They shifted their war on gold to become a war on the Euro,,,, only too late. Now, knowing that the Euro is a fact, we must have a super gold price if the dollar is to stay in the game! The question becomes one of supporting a cheap paper price for the sole function of keeping the market and all its bullion players alive. With the war on gold over, they need to turn their tanks around to face the real enemy but cannot.


cont.

JR said...

cont.

So it seems that as the war switched from dollar v. gold to dollar v. euro, the euro side helped make the dollar gold market TBTF. But with a rising physical gold price/demand, the dollar paper gold market has to keep up because it’s TBTF now. Too many of those “gold” FDIC stickers out there! If those stickers fail, the dollar loses. So the “gold” market is TBTF. Remember this from FOA?

FOA (10/9/01; 10:05:48MT - usagold.com msg#117)

What doesn't seem to be obvious is the "why for" the paper market grew so large. It grew to dominate because worldwide dollar expansion reached its "non-hedged" peak. In other words, the dollar's timeline was ending as its ability to produce non price inflationary economic gains came into sight.

In order to push dollar holdings further, international players needed and purchased "paper financial hedges" to balance their risk. Within their total mix of derivative hedges were found "paper gold price hedges"; modern gold derivatives. The important thing to remember is that these positions are not and never will be used to demand physical gold. They are held to buffer financial and currency risk associated with holding any form of dollar based asset. To work these items don't need to really perform "dollar price movements" in the holders favor as much as they are present in the portfolio to act as insurance stickers.

In that truth, these paper gold positions act like FDIC insurance at our banks. It can and will manage only a small determined portion of bank runs,,,,, not a full scale failure of the banking system. In a real full banking failure we would all get, perhaps, 80% of our covered $100,000 and 10% of the rest.

The same is true for these gold position's performance; real gold delivery along with true price performance, matching real bullion trading, would be only for the very few. For that matter, an actual functioning paper gold marketplace would be for the very few, too! But, in the same way a bank account owner understands the credibility of FDIC insurance when times are good; the international dollar asset owner will not grasp that modern paper gold hedges cannot be allowed to work until after a real serious price inflationary run begins.

For the first time in this portion of the dollar's timeline and our lifetimes, such an inflation is about to show its face!


cont.

JR said...

cont.

So the paper gold of the bullion banks is now TBTF. Of course that doesn’t mean it can’t fail. It either fails, or the USG hyperinflates the dollar as prices rise. They are related, and each will likely cause the other almost immediately, but either one could end up being the initial cause IMO. If price inflation forces the USG to hyperinflate then the paper gold insurance stickers will have to fail to perform. And if these price rises in the gold market fail to manage the flow (demand) of physical as they have so far, we’ll likely see a 10% or larger GLD puke at some point. That would signify more than a 120 tonne allocation demand, a system-busting size. They might think they can rocket the price at that point and get it back, but more likely we’ll see more allocation requests coincident with a falling (paper) "gold" price as the longs dump their worthless “insurance” while wishing they had the real thing.

FOA (06/12/00; 19:48:25MT - usagold.com msg#26)
Put your cards on the table!

The current paper gold world will die (burn) as its value to users erodes, not increases!

…Again, most everyone in the Western Gold bug game is running with the ball in the wrong direction.

…So who is in danger of being hurt as this unfolds?

That's right, the Western paper gold long! I'm not talking about just the US market! This is about the entire world gold market as we know it today. The real play will be for the ones that get out in front of the move by owning physical…

It seems every Gold bug sees only half the trade and has great faith that contract law will favor a short squeeze. Yet, none of them see where it is the long that will be dumping and forcing the discount!

JR said...

Gold: The Ultimate Hedge Fund

hedge fund n (1967) : an investing group usu. in the form of a limited partnership that employs speculative techniques in the hope of obtaining large capital gains

[...]

Hedging

According to Wikipedia the first hedge fund was created in 1949 when Alfred W. Jones formulated a method for going long certain securities while shorting others in order to neutralize the risk of movements in the overall market. He was balancing his exposure to uncontrollable but inevitable cycles.

Today, the big money is all hedged. Almost no one with a sizable account holds only long position bets. The market isn't balanced by 50% betting on one side and 50% betting on the other. It is balanced within each portfolio through leveraged hedges. And it is the evolution of these hedging instruments that has both extended the life of the dollar like a steroid injection and at the same time, sealed its fate.

During Bretton Woods, foreigners held "good as gold" dollars, "the hard currency", as a hedge against their local currency risks. But once those paper gold derivatives we like to call FRNs grew too numerous, all bets were canceled, conversion denied, and those who still held the paper lost out in the immediate devaluation. The same thing happened 38 years earlier... and the same thing is happening 38 years later!

In the 1970's the liberated physical gold market proved to be an excellent hedge against both currency and default risk. Then in the 1980's we were treated to an amazing growth spurt in electronic exchange traded futures and new global exchanges trading these derivative hedges, ultimately netting more than 90 different futures and futures options exchanges worldwide.

In the early 90's, the dollar saw its match as the Euro was taking shape. To counter this threat it promoted derivative hedges as a way of insuring dollar dominance. These hedges, including gold derivatives, only served to leverage the entire dollar system beyond its ability to serve as a real fiat money system. The whole dollar landscape become just a trading asset arena, evolving away from any meaningful currency use to trade for real goods. It can head in no other direction now because our local economy, the US economic base, cannot possibly service even a tiny fraction of the purchasing power currently held in dollars worldwide.

We are now at the "end time run" in fiat dollar production that will soon crush all hedging vehicles. One item alone, physical gold, because it is the main wealth asset behind the next currency system (see: Central Banks), will outrun everything by a wide margin. No matter the derivative's hold on it! Just like gold to the pre-'71 dollar, paper and physical will soon blast off in opposite directions.

Paper Promise Hedges

The purpose of modern paper hedging instruments is no longer to simply balance a portfolio with opposing bets, but it has instead evolved into a risk dispersion game. Like an insurance company, the writers of these instruments issue highly leveraged promises of protection from the risks inherent (and inevitable) in an unstable and unsustainable system.

JR said...

FOA: Lost in all the confusion is the distinction between investing in the price of gold and investing in gold itself.

Anonymous said...

Victory,

on the question of why the special status of China, I can only guess: perhaps in order to cover up the fact that China stopped buying US government debt around the summer of 2011?

On why no QE3 (yet)? Since fall 2011, we have what the financial press call the "Euro crisis". So truck loads of hot (stupid) money leave the Euro zone and move into dollar and pound. This flow of investment capital is a capital account surplus for the U.S. which compensates their trade deficit. Perhaps this is enough to keep the U.S. going for now?

If this idea is correct, we won't get QE3 when the Euro goes below $1.20 (as Rickards used to say), but when the hot money turns around and starts leaving the U.S.

Victor

Victory said...

VtC,

thanks for sharing your thoughts I always enjoy hearing your perspective.

I found FOFOA's response to the question Ari posed thanks to JR's search tip & he crushed it as usual:

"ARI!!!

Where have you been? ;)

I'd like to take a stab at your challenge. Imagine you live in some bizarro world where you're somehow living off of nothing but 37 different credit cards. You've got one big one, maybe a Citibank Card, and 36 smaller ones. You're constantly rolling your old balances into new cards with low or zero interest and expanding your overall debt at Starbucks and many other ultra-hip places every day. Your friends think you're just the coolest thing ever because you're always picking up the tab. And you fully understand that this game will either go on forever, or you'll end up in some sort of bankruptcy. Either way, you'll never have to actually "pay" for anything you bought in the past. Life is good!

Then one day Citibank cuts you off. They can't give you any more credit, but they also don't want you to default immediately. They realize that their weighting in your credit report is as oversized as their share of your debt. What they really want is for you to start paying off your old debt (to them) with new credit from your other smaller cards, for as long as possible.

So rather than announcing (on your credit report) to all your other smaller creditors that your credit has been pulled, they (and you) set up a special direct line of credit to you personally. With this direct line they will help you roll over that portion of your maturing debt (to them) that you cannot immediately pay, into shorter term debt and, to the outside world, this will look as if they are not only still giving you credit, but that they now give you "special credit" through a super new direct IV into your main vein.

But do you announce this new "special relationship" with your long-time largest creditor while you're in the middle of Operation Twist(ing on the gallows)? Of course not! It's not exactly something to brag about. They just cut you off. But you hope that your other creditors will somehow notice it and interpret it in the way that you intended. If, at some point down the road, they realize you actually aren't getting any new credit from Citibank, you can always leak the story to Reuters and hope it gets spun in the way that benefits you. After all, this 37 credit card scheme has been working pretty darn well for a really long time. So no real worries, just same ol' same 'ol.

Sincerely,
FOFOA

PS. I will note (from my last post) that those smaller credit cards ponied up $68B in "surplus" credit at the same time as China reduced its balance by twice that amount (beginning in June when the "special relationship" began). "

burningfiat said...

Enough, thank you for the call to get out. I've been enjoying the day with garden work!

Hi All,

I would like to bring up a general topic for discussion. Mainly for the benefit of newcomers:

Does anyone think the IMF accounting rules on gold vs. gold receivables need to be changed for Freegold to emerge? Or can anyone argue that the IMF rules are inconsequential to the emergence of Freegold?

Thanks in advance,
/Burning

JR said...

Accounting rules = monetary plane number mumbojumbo

The #PhysicalPlane Is What Matters!

I'm also not really interested in the flow of dollars within the monetary plane of "investments". Investments within the monetary plane change price regularly, sometimes with great volatility, yet without crashing the entire global monetary and financial system. But that real stuff in the physical plane, stuff like food, energy, medicine and industrial inputs, is (remarkably) relatively stable on its dollar price tags over time (at least compared to currencies going through hyperinflation). So we don't need to picture the dollar flow as a portion of the entire dollar stock, we can instead picture it as a flow of real goods and services as long as we focus on the goods and services portion of the BOP. And we also know that government spending (the federal budget) is all on goods and services in the physical plane, not on "investments" in the monetary plane.

So what do I mean by "marginal flow"? The US is the dollar's home, its creator and its legal tender zone. Most everything here carries a dollar price tag. But the US also trades with the world outside of its own currency zone, and in so doing it emits dollars. Last year the US spent $2.66T abroad, but we also took back most of those dollars by selling our stuff abroad. In fact, we took back $2.1T of the $2.66 we sent out. So netting it out, we net-emitted $560B last year. That's 560B dollars created here in the dollar creation zone and sent out into the non-dollar (homeless dollar) zone. That's marginal (deficit) flow. But there's more.

Before I get to the "more", let's reduce this to an easier time-frame. In a stable currency (like the dollar), the prices of necessities like food, energy, medicine and industrial inputs don't change much over a one-year time period. But prices can change overnight, and that's what I'm predicting. So I'm going to start quoting these annual statistics in daily flow amounts, by dividing the annual number by 365. That, of course, includes weekends and holidays. And while our beloved monetary plane closes down for weekends and holidays, the physical plane of necessities runs 24/7.


Inflation or Hyperinflation?

JR said...

"I cannot see a dollar collapse without a simultaneous revaluation of something else. It's a seesaw. The dollar isn't collapsing against gold. It is collapsing against the physical plane of goods and services. That's the fulcrum, not gold. Dollar collapse is the force, goods and services the fulcrum, and gold the load. So gold is revaluing against goods and services. The gold revaluation is against the physical plane so as to fill the reserve void left by the dollar's collapse."
Unambiguous Wealth

burningfiat said...

Thanks JR, Much appreciated perspective!

JR said...

Moneyness

...the act of government deficit spending without either counterbalancing taxes or Treasury sales to the private/foreign sector, and the act of Fed quantitative easing, both change the nature of the money supply in a way that all other "normal" activities do not. They debase "our money" by expanding "their money" in volume to ease their discomfort. And this kinda gets us to the driving thrust of MMT; that MMT sees little to no danger of this monetary plane debasement spilling out into the physical plane with deadly consequences for the dollar.

There is, however, one area in which the danger is at its all-time peak today. And that is the US trade deficit as viewed from the physical plane. But before we get into that, let's take a look at a couple neat charts that Cullen Roche uses to visualize the monetary plane accounting identity that underlies his theory. Cullen calls them "sectoral balances," meaning the monetary plane balance sheet of three different sectors: the government sector (USG), the domestic private sector, and the foreign sector.

What I'm going to try to do is to help you see the physical plane reality of these charts. They are so neat and balanced in the monetary plane, yet they represent an immense imbalance in the physical plane that, because of the credibility inflation of the last 40 years, leaves the dollar vulnerable to spontaneous hyperinflation.

[...]

The Debtor and the Junkie

The USG may be a dealer in the monetary plane, but it is most definitely a sketchy junkie in the physical plane. The USG thinks (and truly believes) that the key to rejuvenating the US economy is trashing the dollar as a short cut to increasing exports (reducing the trade deficit). But what it can't see (nor anyone that focuses solely on the monetary plane for adjustment) is that the huge trade deficit the USG wants to quit is actually its own heroin fix. This is a deadly combo for the US dollar.

[...]

What I'm going to show you is that there's something quite dangerous to the dollar that is already well underway. From an accounting perspective, there's not much difference between QE and the easy money prescriptions coming from some of the MMTers. And these seemingly innocuous "fiscal operations" are actually born of necessity arising, not in the monetary plane, but from the physical trade deficit.

Unfortunately, the USG/Fed believes that trashing the dollar will help the domestic US economy as a kind of short cut to growing exports and thereby increasing wages and consumption demand. In other words, if we could just make our products cheaper overseas through monetary plane operations, we could sell more real stuff and thereby we'll have more money and all will be peaches and cream.

But the problem is that, net-net, the US consumes everything it produces and then some. This intractable problem cannot be solved in the monetary plane, except through dollar hyperinflation!


cont.

JR said...

cont.

[...]

The bottom line is that the USG cannot crash its own lifestyle. And when the dollar starts to "sink", that pile of pennies in the video above will be insufficient (not enough money). Luckily, that pile of pennies represents the budget of the currency issuer himself. So he’ll just increase it, to defend his lifestyle, while scratching his head at why the trade deficit has nominally widened rather than narrowing as he thought it would when he trashed the dollar.

One of the strongest arguments that the USD will not hyperinflate like Weimar or Zimbabwe is that the USG's debt is not denominated in a foreign currency. If it were, this would be a different kind of hyperinflationary feedback loop we were facing. If all the USG debt was in a foreign currency and the dollar started falling on the foreign exchange market, that debt service would lead to hyperinflation. But that is not the case. So it’s not the FX market (monetary plane) that is the big danger to the dollar.

The dollar is the global reserve currency, so it is the physical plane that is the biggest threat to the dollar in the same way the FX market was a threat to the Weimar Mark. And it is not the nominal debt service that is the threat like it was in the Weimar Republic, but it is the structural (physical plane) trade deficit. To the USG, that is the same threat as nominal debt service denominated in a foreign (hard) currency was to Weimar Germany.

As the German Mark fell, there was "not enough money" to pay the debt. And with a little inflation, there is "not enough money" to buy our necessities from abroad.

Texan said...

AD,

It's the opposite, because gold is infinitely supplied. It's never used up. But just try and go and buy it, in size. You can't. The only way size demand is filled is through "something else". Ie, paper gold. GLD. Etc. The use of paper gold expanded supply to meet demand - but it isn't gold, it's a contract. It's a fiat-settled contract in extremis (most likely). Do you see? As for why entities buy paper gold, no doubt a lot of them think it is close enough, like the liquidity, just trade price. Who knows. But without paper "supply", gold would be more like London real estate. And no doubt in extremis many paper gold buyers will wish they had bought physical if delivery ever fails.


Corn - or any commodity - is totally different. Supply must find demand. If there is a bumper crop, that crop bids for cash, and vice versa - and more critically the cash looks for substitutes if prices move higher. Corn too high? Buy soy. Can't afford soy? Buy hay. Oil is the same- if price moves too high, buyers try and sub it out or reduce usage. Point being, commodities are all used in processes of some sort or another. Eating, energy, building, etc.

Gold isn't really used for anything other than SoV bullion or SoV jewelry. So how exactly is the price set? It is set by artificial sales. Paper gold. And because it isn't used, this works for now. "I have a contract" is no different than "I have a safe deposit box filled with bullion" to many, many people. As it would be, because by and large people still believe in the sanctity of contracts. But it isn't real physical gold.

Try the following thought experiment. Imagine they closed all futures markets everywhere tonight forever. How would commodity prices be set? Well I can imagine a bunch of seller agents and buyer agents out there trying to find each other and establish the best price. What about gold? Well, 3,000 tons would have seller agents. But not most of the other 175,000 tons. It isn't for sale. So what happens when buyers come in to buy 7500 tons? Who do you think sells that?

Do you see? It can only be supplied now through paper promises. And "additional physical supply" only comes available at higher and higher prices ( or rather, when the lure of "paper contract returns" exceeds the expected opportunity cost of holding gold, e.g. 20% USG bond rates in early 80s backed by a "believable" promise to hold currency more or less stable"). Put another way, without paper gold supply there is basically no way to buy physical gold.

JR said...

Key:

So it’s not the FX market (monetary plane) that is the big danger to the dollar.

The dollar is the global reserve currency, so it is the physical plane that is the biggest threat to the dollar...

Victory said...

db,

thanks for noticing and taking interest, good to know I'm not alone.

I here you about HF managers being sharp, most of them, but there's a difference between being really smart and being on the cutting edge of forward thinking and having the courage to stand behind the conviction of your research/analysis with millions/billions of client funds. I think two things are in play here.

1. There aren't to many HF managers that posses the kind of smarts, nerve, and capital needed to be a catalyst, SWF's not-included. A dozen? two dozen maybe? and they would have to move as a group, I don't think one or two wouldn't be enough.

2. As as you eluded to, maybe they have the smarts and the capital but not the nerve, or the nerve but not the capital, ect..

Kyle Bass put his money (clients money) were his mouth is but how many Kyle Bass' are there, again I think as intelligent as the HF community is there is a lot of group think within this bunch and they move in herds and following momentum. Few have the courage to lead ahead of the pack as the old mantra goes 'the markets can remain irrational longer than one can remain solvent.'

Have you ever read Michael Lewis' book the Big Short? This physical vs. papergold market distortion reminds me of the CDS mispricing of CDO's which was the theme of the book. There were a few HF's who bought CDS because they saw the ponzi for what it was (two of the books main characters Steve Eisman and Dr. Michael Burry) took a lot of heat from their clients as they held on to the their losing CDS positions waiting for the eventual pay-off. Dr. Michael Burry had to freeze client funds, actually preventing redemptions - in the end he was vindicated because his CDS position payed off BIG TIME but even though he made money for himself and his clients he notes that his clients were still mad about the look-up provision, so yea I agree client concern is a major issue, at least for domestic HF's maybe not so much SWF's. I think SWF may be more concerned with the longer-term consequences of of their actions whereas domestic HF's are more short-term profit motivated.

In the book it wasn't the group of forward thinking HF's who stressed the CDS/CDO market - it wasn't until the Primary Dealers (the Big Boys) started turning on each other that set the spark off.

-v

Jeff said...

ECB is pumping air into the Bernank's bubble. POP!

MEXICO CITY (Reuters) - Cutting the deposit rate the European Central Bank offers lenders in the euro zone below zero is an option, ECB Executive Board Member Benoit Coeure said on Friday.

http://finance.yahoo.com/news/ecbs-coeure-says-negative-bank-035738110.html?l=1

FOFOA: Two Historic, World-Class Bubbles are About to Pop

Bubble #1: Government Debt (with a nominal value in the tens of TRILLIONS)

Bubble #2: Perceived Wealth, denominated in purely symbolic, un backed, unsustainable-Ponzi-debt-based currency (with a nominal value in the HUNDREDS of trillions)...

It is the 28-year hyperinflation of the US$-denominated debt asset bubble that is about to pop...

But the thing about THIS bubble's rise that is so different from any rise in gold is that the price of past issued debt has a natural upper limit. And the "lowering of interest rates scheme" has a physical floor, an inevitable and unavoidable dead end... call it ZERO.

Yes, we have seen a couple ventures into negative interest rate territory lately. But this is simply anathema to the very concept of money, period. It is the ultimate froth, the last breath of air you can blow in before a bubble pops. It is the sure signal that the end is nigh.

When interest rates hit ZERO, they only have one way to go. And that means that the value of past issued debt, the very kind of TRASH that China is sitting on a land-fill mountain of, only has one way to go... DOWN. This is the very definition of a bubble that is about to pop. As Peter Schiff calls it, this is The Mother of All Bubbles!

Anonymous said...

How safe is a money market fund?

Fed eyes limiting withdrawals from money market funds .

The analysts propose that money-market funds could be strengthened if they were to have a "minimum balance at risk." As envisaged by the authors, this balance "would be a small fraction of each shareholder's recent balances that would be set aside in the event that they withdrew from the fund," the press release said.

While regular transactions would be allowed as they are now, this special minimum balance would be locked up for 30 days. "The delay would ensure that redeeming investors remain partially invested in the fund long enough to share in any imminent portfolio losses or costs arising from their redemptions," the bank explained.

Jeff said...

http://www.zerohedge.com/news/t-30-days-10y-treasuries-yielding-less-1

FOFOA: In imagining the dollar being "loved to death" I'm picturing crowds piling into a bomb shelter ($IRX) for protection, and once they are all in, the shelter itself blows up. They never have a chance to rush out into something else. Simply stated, all something else's drop their bid for dollars...

In other words, it will appear on the surface (very briefly) to be a deflationist's wet dream, but in reality it will be anything but that. And it will happen so quickly and with such confusing signals that most people will simply be trapped in their positions. And the few traders with cricket-like instincts (but no understanding of what is happening) will likely jump into the fire rather than away from it. And then, when the dust settles, a new price discovery market for gold the elemental metal will eventually make itself known.

Aquilus said...

I was going to do a post for newcomers to expose AD's logical fallacies (so they don't stand unchallenged) but thanks to today's great posts I don't have to.

First off, in case you were thinking you could acquire some "insights" from AD: we have a direct quote from him: You are correct that I cant prove anything, neither can you. . So don't bother looking for knowledge from that source.


On that, "neither can you." part, have a look at the items below (plus the links):

1. What Texan said (great comment Texan!)

2. Here's JR from Twitter today :


From that LBMA survey, we can see that the LBMA had net sales in one quarter of 7,575 tonnes of paper gold.

Paper gold=most likely FOREX use of gold as a hedge/currency play, but BBs still have to hedge price exposure

Without a hedge, that would be a 7,575 tonne naked short position for the BBs GLD Talk Continued

That net increase in paper gold is also a net inflow of cash for the BBs, which they use to hedge the exposure

Cash inflow to BBs was $338B. That’s over 3 months, so it’s @ $5.4B per day inflow. GLD Talk Continued

5.4B = a small % as the daily turnover in paper gold is $240B and the daily turnover of all currencies is $4T.

So in a $4T/day FOREX market, that’s a $5.4B/day net flow from other currencies into gold. GLD Talk Continued

There’s no way they hedged all of that in the "gold" market (Comex/mining forwards/GLD). It’s not big enough

BBs must be hedging this exposure the way they hedge net positions against other currencies in the FOREX market

BBs can also hedge some of that exposure in the gold market and try 2 stretch physical and replenish a GLD puke

BBs have some $PoG control, but r still relying on physical side to act as expected. GLD= finite reserve pool


One last note: he confuses the GATA/King World News/Hard Money Socialists with readers of this blog when he says:

Isnt it amusing that so many jerks around the world shout, that "paper food" is supposingly raising the food prices for the third world? But for the "GATA folks" it's the opposite? Looks like everybody sees what he wants to see

Again, look at Texan and JR's notes above, read the linked post, and if you REALLY want to be educated, go directly to the source: read FOFOA's posts. Here are a few must-read posts, but that's just to whet your appetite: JR's suggested reading list in PDF format compiled by MatrixSentry. Enjoy!

costata said...

Aquilus,

100 per cent agree. Great comments and discussion. I especially liked Texan's breakdown and comparison. And the highly relevant extracts from JR and Jeff.

I'd like to throw an idea into the mix. After the transition the CBs will be openly backstopping liquidity in the gold market. Ready to step in at any time necessary to act as market makers. Could hedging by gold miners be addressed via the FX market? In other words would commodity style gold markets like Comex simply become unnecessary.

The price volatility in gold should be minimal over short-to-medium time frames. The long term trends in currencies would be playing out in BOP time scales. Would the FX market meet all of a gold miners hedging requirements?

Chico_hawk said...

Has it ever been definitively established where Europe's gold reserves are?

Obviously, much of it is in Europe, but the last I saw, much (most?) of it was still held in New York (& London).

With all the (correct) talk about not owning anything unless you actually possess it...just wondering whether Europe has possession of even most of its gold or is relying on contractual (paper) agreements to lay claim to it...

If the $US enters hyperinflation, I can imagine the US may look at that Euro gold in a different light...(if they haven't already sold it on the market to meet physical demand to keep the $US ponzi going...)

Comments?

costata said...

Chico_hawk,

Some priorities transcend short term issues. FWIW I think that fears about the US grabbing the gold at the Fed owned by foreign countries may be overblown. Even during wars some eyes are focused on the post-war period. But that's just my 0.02

KindofBlue said...

FYI

Jim Sinclair has a formula for valuing gold (he divides the total amount of US Treasuries held overseas by the total ounces held by the US). His current back-of-the-envelope calculation puts the price at $19,533/oz.

http://4.bp.blogspot.com/-e_aVL-qManI/UAvynUUB1hI/AAAAAAAAH5Q/U4YeZYsThBc/s1600/FDHBFIN.PNG

When he originally ran this calculation in 1974 he predicted then a price of $900/oz. which it nearly hit in 1980.

I share this for noobs who may not know this, but what is more interesting is I was wondering if anyone else here is catching that breeze from Genoa? (smile) Seems to me to be a simular understanding of the function of gold.

Cheers

Chico_hawk said...

Further to my previous post...

http://www.spiegel.de/international/germany/debate-breaks-out-in-germany-over-foreign-gold-reserves-a-833289.html

Ironically it is the "austere" Bundesbank that is the entity that wants to keep most of Germany's gold in New York AND doesn't want to allow Germany's governmental auditors physical access to that offshore gold so they can verify that it in fact is all there and instead the Bundesbank simply wants everyone to take their central bank counterparts in the US (NY Fed) & London's word for it...

With all the scandals (Libor, TBTF bank bailouts, etc.) taking the Fed's word is hardly something to be reassured by.

costata said...

Loved this quote that Richard Russell used to introduce a piece about the RE market:

"Some problems are so complex that you have to be highly intelligent and well-informed just to be undecided about them." Dr. Lawrence Peter through courtesy of Marc Faber.

http://www.financialsense.com/contributors/richard-russell/housing-shows-bullish-signs

costata said...

Nearly 50% of the total outstanding debt of the world's top 10 debtor nations needs to be rolled over by the end of 2015.

http://www.financialsense.com/contributors/chris-puplava/global-qe-coming-let-gold-mania-begin

In my somewhat less than humble opinion the countdown starts January 2013 and we will be able to look back from the end of 2015 (at the latest) and finally put to rest the arguments about whether A/FOA were right in their macro analysis.

costata said...

I sent this to our favourite Yeti back in November 2011. I think it's a good counterpoint to the sovereign debt numbers and rollover timetable that Chris Puplava is discussing in that link above.

I wrote:
This piece contains most, if not all, of the mainstream arguments in support of the claim that ultimately the ECB will print (and with Germany's enthusiastic suppport no less according to this writer).

This piece is also important for this reason:

The stunning success of BHP Billiton’s bond issue last week is a clear demonstration that these days global investors prefer corporations to governments, managers to politicians.......

..... Last week BHP raised $US3 billion at interest rates ranging from 1.2 per cent for three years to 3.25 per cent for 10 years – less even than the Australian government pays for debt, and it’s a solvent government.


If you have any confidence in Martin Armstrong's work on cycle theory then this debt raising is the poster child for the shift from confidence in public institutions to private ones.

http://www.businessspectator.com.au/bs.nsf/Article/eurozone-debt-crisis-euro-currency-money-printing--pd20111121-NSRJQ?OpenDocument&src=kgb

Now that last paragraph may be a bit over the top but I think this debt raising may have been a bellwether.

JR said...

Hi Chico_hawk,

you queried:

"With all the (correct) talk about not owning anything unless you actually possess it...just wondering whether Europe has possession of even most of its gold or is relying on contractual (paper) agreements to lay claim to it...

If the $US enters hyperinflation, I can imagine the US may look at that Euro gold in a different light..."


Check out FOFOA's 4 part comment to "Moneyness" and followup discussion in Discussion Forum.

Here's a snippet from FOFOA:

Rickards predicts the USG will take Germany's gold as a response to the collapse of the dollar. Does this make sense in light of the USG's needs at that time? The USG won't need gold or money. It will need continuity in a trade inflow of the real goods and services needed to run a large government and military.

[...]

When the USG called in the gold in 1933, it was legal tender currency that could be called in.

[...]

you suggest that the big bad USG will simply take the oil when it can no longer buy it, and it will take Germany's gold per Rickards. So what if it does? Do you think either of these actions would reverse a dollar collapse or Freegold revaluation? Would stealing oil and gold make for a viable "strong dollar policy?"


Cheers, J.R.

JR said...

How Can We Possibly Calculate the Future Value of Gold?

Stefan Pernar said...

Have been doing a bit of math:

Total world wealth in 2007 in 2000 US$ = 125.25 Trillion [1]
Equivalent 2010 dollars = 125.25 TUS$ * 1.25 [2] = 156.56 TUS$
Total above ground gold = 158'000 metric tons [3]

Theoretical maximum gold value assuming ALL wealth flows into gold = 156.56 TUS$ / 158'000 tons = 30'816.55 USD / oz


Hello Stefan,

I don't think it is as simple as pouring a hundred cups of water into one large beaker and noting the total volume. But you do bring up a few common misconceptions. There are a few trillion possible methods to attempt to presage the full impact of this thing called Freegold. So as I said in Metamorphosis, "Let's spin this globe and take a look at things from a slightly different angle."

Instead of looking at wealth, or even debt, let's look at "purchasing power". Better yet, let's look at the concept of "Stored Purchasing Power". Now I realize that most people's "stored purchasing power" will be deployed at some point over the next ten or twenty years... those that have any. But for the sake of understanding the theoretical concept, imagine that I have immense stored purchasing power. Imagine that I am a "super-producer" giant. Imagine that I make something that everyone in the world wants and needs.

[...]

You see, time is the factor most ignored in the concept of "stored purchasing power". It is ignored because it is relatively irrelevant to most people. This is perfectly understandable. But does this mean that I should forfeit the fruits of my labor after some point in time or at some maximum? Of course not! That would be socialist nonsense. As long as my storage of wealth medium does not infringe on anyone else's industrial growth, then my accumulation actually contributes to economic expansion.

The future amount of time is infinite, therefore "stored purchasing power" is theoretically limitless. The only thing that limits its potential is a faulty storage medium, which limits the collective confidence in its ability to preserve wealth over time.

With a faulty storage medium I will not be as eager to store the fruits of my labor for deployment so far into the future. For I will recognize that at some point in time the medium will fail and my efforts will have been for naught. So I will be more likely to "spend" my considerable wealth in the here and now. Not right now, but you know what I mean. I'll probably build a 70,000 sq. ft. high tech castle on a lake for me and my wife and things like that.

And an interesting side effect of spending my considerable wealth in the here and now is that it not only reduces the purchasing power of the rest of my wealth, but also everyone else's who holds a similar medium as me. In aggregate, a faulty storage medium is self-limiting.

So, quickly cutting to the chase, the logical conclusions we can deduce from this conceptual line of Thought are that:

1. the storage of purchasing power is size-unlimited in a solid medium with potentially infinite confidence and one that does not infringe upon anything else, and

2. the storage of purchasing power in a flawed medium with a mathematical limit (like debt) is constrained roughly to the aggregate purchase price of everything in the world at any point in time, with a decent margin of error.

Nickelsaver said...

The very idea that AD's comments require countering by anyone on this forum is laughable. He is a running joke; a punchline without a laugh.

Anyone that wants to read his exhaustive rebuttal to this site can do so <a href="http://antifreegold.blogspot.com>here<a/>.

He has been working on his thesis for many months, so be prepared to read thru a lot of material, not.

Nickelsaver said...

whoops, link fail...not worth repairing...lol

Tony said...
This comment has been removed by the author.
Chico_hawk said...

Thanks JR

Will review that again.

costata said...

I came across this in Doug Noland's latest newsletter:

July 19 – Bloomberg (Charles Mead): “Yields on U.S. corporate bonds of all ratings fell below 4% for the first time as Europe’s sovereign-debt crisis boosts demand for U.S. assets deemed safer even with a slowing economic recovery… The gauge was at 4.81% at year-end and 4.59% a year ago…”

http://www.prudentbear.com/index.php/creditbubblebulletinview?art_id=10687

costata said...

For Aussie readers and China watchers, I think the writer absolutely nails it here.

http://www.macrobusiness.com.au/2012/07/healing-china-means-hurting-australia/

Let me say up front that I’m a long term China bull. The production base that China has seized in the past decade is the stuff of super powers.

Perhaps well worth any losses from stepping up to buy USG paper and kick the can down the road.

..If managed right, it means a reasonably smooth internal transition from fixed-asset driven growth to consumption led growth...

... For Australia, the lesson is simple. Healing Chinese imbalances is bad for Australia’s chosen reliance upon a small set of building block commodities. China will prosper but we will face our reckoning.


(My emphasis) Damn right IMHO.

AdvocatusDiaboli said...

Texan,
I appriciate your thoughts, helps to look at it from also other perspectives. Let me just point out the issues that are a little questionable:

"But just try and go and buy it, in size. You can't."
Says who? Really, how do you get to the idea, that this is true? (Please look at the old-coin-phenomenum I described earlier) I am not saying that it is wrong, just wondering what comes to my mind that e.g. Mr.Chávez got his gold without any problems.... And again, that's the same argument always heard from the Physical-Silverbugs. Guess we agree that they were wrong (although the physical silver supply appears much smaller to me and is another show anyway, just saying as an example), so why are you so convinced about that one?

"But without paper "supply", gold would be more like London real estate."
Yep, are we now talking about, that it would tend to be more "bubblish"? I agree with your argument on that one, but in that comparision you probably would also agree that it would not be the predicted FG waterfall, but rather a classical bubble.

"Gold isn't really used for anything other than SoV bullion or SoV jewelry."
Looking at the stats this is not 100% true. From the mine supply ~12% go into industry. From a supplier I got beginning of the year a letter: Due to rising gold prices we have to increase our prices with 10% gold extra...
And as I described my personal gold stash philosophy: You can consider me a landfill today and at minimum as long as the € exists and no crooks hand from the street lamps. Has absolutely nothing to do with the 20bagger prices potentially offered to me.

"Try the following thought experiment. Imagine they closed all futures markets everywhere tonight forever."
Yes interesting thought: I assume that the cash4gold&coinshops shops in your street will still be open and the same people will enter like they did the day before? Probably the mines will also try to continue to mine those yellow stones and will try to find somebody to sell to.
And now about the "Waiting-for-Godot-paper-suppresses-gold-bugs", can you tell me where he get's that great deal in real life and where he can verify a "awesome" price? At least as a "mini-giant" I wouldnt pay him for some marples those prices (because I already have far more over reasonable marginal utility).

"Put another way, without paper gold supply there is basically no way to buy physical gold."
I guess I can agree about that one. And how about the other way around: Without the paper gold market, there is basically no way (to know at what price) to sell physical gold?

Again, I can completely understand those arguments, but still lots of assumption on one hand and one the other hand no reasonable describtion of a market that would reasonably consider that "above stock".
Greets, AD

Candy Sange said...

AD said: Without the paper gold market, there is basically no way (to know at what price) to sell physical gold?

FOFOA said: During this time, after the paper market has failed, that GIANT sucking sound you hear when you call your dealer and mention that you have some gold for sale will be the CBs and Giants somewhere at the other end of the dealer network with their unlimited currency, their insatiable demand for gold, and their standing over-bid acting like a giant concubine sucking a golden golf ball down a tiny hose. Let's call these Giants and CBs "the buyers of last resort" for gold. Another said they stand ready to buy any and all physical gold offered for sale.

Candy Sange is out [buying physical gold at the coin store]!

AdvocatusDiaboli said...

Candy,
"because FOFOA said so", appears like a great argument to me....
Remember FO(FO(A)) were dead wrong with other predicsions, but still just taking blindly their word for it, without any other thought or reasoning?
Greets, AD

Candy Sange said...

Blah blah blah AD.

They said it and that doesn't automatically make it so. My thinking about it and becoming convinced myself that it's The Truth, when I place it next to everything else I see happening around me, is what does it for me.

So you have a great day there, below sand level.

Texan said...

AD,

It's hard for me to go into great lengths responding due to the limitations of the blogger commenting functionality, but I will try.

I don't know that gold isn't for sale in size. I thinks it's highly likely that it isn't though, because all official sales have pretty much ceased (as far as I know). And even when they sold, they never offered more than a few hundred tons. And I have no idea if that was even physical, or just allocated/unallocated somehow. Plus, the increase in price doesn't suggest more supply coming into the market.

Yes ilondon is in a bubble. Every SoV is in a bubble. But a bubble due to what? More printing. Wealth has to find something to transport itself in, and many are choosing London (for a variety of reasons, not just inflation hedging). My point about London real estate ws simply that it cant be captured via a paper market. A physical buyer and physical seller meet and transact. And because taking title can be so onerous, no one wants a "derivative" or "promise" to take title at some later date. They want the keys.


I don't think there is going to be a transparent market for quite some time in gold absent futures, but my guess is the shift will be to Asia/India to look at cash prices being paid.


Here is kind of the summary if you will. Gold is basically the "anti-promise". It has no counterparts risk. So it is axiomatic that one cannot buy this through....promises (I.e. contracts). And short of London real estate, art, maybe some farmland, etc......all of which have multiple issues as SoV, what else is there if you don't want counterparts risk (where fiat also has counterparts risk due to continually expanding at unknowable rates)?

Jeff said...

IMF attempts to force ECB to fund Greek profligacy?

The International Monetary Fund will stop paying further rescue aid to Greece, making the country’s insolvency in September more likely, the Der Spiegel magazine said. citing unidentified European Union officials.

http://www.bloomberg.com/news/2012-07-22/imf-to-stop-further-aid-tranches-to-greece-spiegel-says.html

FOFOA: The euro is behaving perfectly predictably in maintaining the nominal performance of its system through expansion, but it cannot be forced to fund the future government profligacy of the PIIGS through volume-only expansion. That link is severed.

Jeff said...

RE is a bubble, stocks, bonds, but gold is never a bubble. It's more of an unbubble.

FOFOA: Most recently there was "the Housing Bubble" and before that, "the Tech Bubble" or "Dot Com Bubble". In 1929 we had "the Stock Market Bubble".

In all of these bubbles demand quickly overwhelmed supply in a feedback loop whereby new demand joined the stampeding herd causing the price to rise, causing more new demand to join the stampede and so on. A self-reinforcing feedback loop. Then leverage or credit was added to the rocket-fuel mixture boosting prices to even higher stratospheric levels. With cheap credit even the shoeshine boy could get in on the action!

And with the shoeshine boy in, new and improved Ponzi-paper derivative financial trading products were created allowing the hyper-acceleration of speculative trading. Accelerated paper trading created shortages in the underlying object of desire which in all the above cases was something that could be produced to meet demand. But these temporary shortages blasted the price even higher drawing more lemmings into the stampede.

Of course there were subtle differences in each bubble. But the commonality was that the underlying object of desire in each case could either be easily ramped up to meet demand through a little extra human effort [2], or it could at least be valued objectively through common metrics like income, rent, earnings, interest or through careful valuation of its component elements.

But changes in the gold price are mostly driven only by investment demand. Industrial supply and demand in gold is very stable relative to investment supply and demand. So any significant rise in the price of gold is a clear indication of growing investment demand and is also a positive confirmation of the premise behind that demand, that gold will rise. This creates a self-affirming feedback loop of positive reinforcement.

And since gold production cannot be ramped up to meet demand like it can in bubblicious items, there is no reason for gold to fall back. Gold mining does not debase gold in the same way that dollars, tulips, homes, Dot Com IPO's or government bonds are debased through production. Mine production is taken from known reserves that are already valued, owned and traded, and all gold on the planet Earth is a fixed amount, the same fixed amount it was a million years ago. All we do is move it around, like poker chips on a table, to those savers that value it the most.

Furthermore, the price of gold is completely arbitrary. This means that gold can go as high as the people of Earth want to take it without EVER exceeding objective valuations by common metrics like earnings, interest or the sum value of its component elements. Gold IS the element.

And finally, as gold's role in the world evolves from barter item to pure numéraire, to transactional currency, to simple commodity, and ultimately to wealth reserve par excellence, its value relative to the rest of the world can (and has) shifted both up and down by as much as two orders of magnitude. Such phase transitions in the functional value of gold completely invalidate "fair value" methodologies like that used by Paul van Eeden.

costata said...

DP,

Eddie Vedder hasn't done nearly enough recreational pharmaceuticals to do justice to Jim Morrison but the band still have their shit together, by and large, methinks.

http://www.youtube.com/watch?feature=endscreen&v=n23YU6dFBsE&NR=1

DP said...

costata,

What Jeff said!

http://www.youtube.com/watch?v=wjHPwdHR_Wg

Aquilus said...

Just a comment for anyone trying to gleam any nuggets of wisdom from AD's latest:

1. He counters LBMA market demand with coin shop demand and supply. Q1 published demand for 7000+ tons from LBMA directly. It's like comparing oil demand at the world level with the demand at my local gas station!

Then the conspiracy of the old coins... Tell me, who in the Western world keeps an asset that that in currency value? Only those who understand value outside of its currency numeric value. Last I checked, most coin shop Giants like AD were not part of that group. Of course people would sell, especially with media calling for gold at$1200/oz

2. AD it's the ultimate "because FOFOA said so" it must not be true. His point of view is still very much that pic the little oppressed ant. But he does own Au, so he'll do ok. If he doesn't sell when paper gold price goes under production price :-)

Aquilus said...

Previous comment: of the little, not pic the little.

AdvocatusDiaboli said...

A.

"Q1 published demand for 7000+ tons from LBMA directly"

so what's about that magical number of 7000t?
Has it been bought, by whom from whom, because you can only buy it at that price if there is a seller at that price (and is there a seller at those supposingly "small" prices...)? Has it been called for delivery? Or (who) accepted cash settlement? On the other hand others argue, that it is impossible to buy larger quantities... Can somebody from the FG-advocates finally make up his mind? In this context, I find this word "demand" kind of funny...

About the "old coin storry" (at least something that is real and verificable), that's no conspiracy. Just wondering through which hands of buyer vs. seller those went. At that amount over a longer period of time of purchases from different sources, statistically it is significant,... And in terms of "investment demand" I guess coins show at least how many shrimps get/are involved (not good, not good at all).

Interesting to follow the amount of coins sales from the mints....yes, look at the years 1975-1985 for the Krügerrand ...and compare with the recent years...
At least those numbers appear to be real. Much more real, than all that EricSprott, KWN, GATA stuff, where people just wanna draw you into something. Or that royal BS about the EuroFreegoldArchitects....
Greets, AD

Victory said...

yet another Gold ETF article confusing resumptions for lack of demand

http://economictimes.indiatimes.com/markets/commodities/leading-gold-etf-sees-biggest-weekly-outflow-of-2012/articleshow/15107887.cms

article notes GLD saw it's largest weekly redemption of the year last week

-v

Texan said...

AD,

I am having trouble following - is your main contention that paper gold sales have no effect on physical demand since the pape supply is just filling "paper demand"?

Sort of like a betting market like Intrde or sports?

If that is the case, I can see your point - but don't you think it's a bit out of whack to assume that ALL paper gold demand is just buying contracts for the sake of owning contracts? Don't you think at least some of the buyers really think they have "gold" when they buy GLD or buy gold futures?

I think it's perfectly reasonable to state tht some portion of paper gold sales re purely to "financial demand" which are just betting on price moves nd have no intention of ever taking delivery. But it's also reasonable to assume some of the demand thinks it is buying physical in some unallocated fashion. In which case such gold sales must be suppressing the price by selling gold they don't have (knowingly or unknowingly, it doesn't matter). People who think they are buying "no counterparts risk physical" are buying "delivery risk" instead. Loo look at UTimCo for example first buying unallocated, then ultimately taking delivery if I have my facts correct. And Chvez! Bit by bit, iN ways publicized and not, buyers are realizing hey don't have gold when they enter. Futures contract. But not ll buyers - not yet.

Point being, if and when everyone demands delivery, do you seriously think everyone in the chain can deliver physical? I m pretty ure th everyone who is long could take delivery - cash is plentiful. But really, can everyone deliver? That is the crux of the issue.

Texan said...

To clarify, I am talking bout institutional and CB money - not retail/coin shops, which are the tiny tip of the dog's tail.

Anonymous said...

Texan,

I think you're right, AD is basically saying that paper supply (unallocated gold) is taking care of paper demand.

I think you're also spot-on in pointing out that AD is assuming that all of that paper demand is just for the sake of owning contracts, which is much more likely to be a false assumption.

I think Bron is also of the similar view as evidenced in this thread , where he says:

The question then becomes what percentage (by ounces) of unallocated account holders are aware and which aren't. Any industry players/commercials would be and maybe some hedge funds who only care for their (as Victor says, leveraged) cash returns, those who blogger FOFOA calls the Giants. You feedback would indicate a fair if not all of the HNWI are not. They may be less industry ones than HNWI, but industry would hold some big balances relative to HNWI. We have no way of knowing but I'd guess that it isn't a case that 95% (by ounces) don't know, which is what most people think I bet and the basis on which they believe a "run" for physical will bust the system.

I'd just say the percentage who know full well what unallocated is and thus who don't care about physical delivery and happy to take cash settlement is much higher than many believe. If true, this means the system has more robustness or less risk of a run for physical and thus the game can continue for longer than many think.


I think AD even questioned a few comments earlier whether unallocated even exists, LOL. It does and here's Bron on usage of unallocated, from the same thread linked above:

Victor touched on some of the advantages of unallocated, being private, lower costs, ability to use as collateral and customisable leverage. To that I would add that trading OTC is far more flexible. You are not stuck with set futures expiry dates or option strikes etc, when dealing OTC you can do a forward to a specific date or choose a specific option strike price and date and the bullion bank will price that custom trade. For a seriously large trader this is better than dealing with the (necessarily standardised) exchanged traded contracts. Any bullion bank will have desks across the globe, so 24hr trading is possible.

For the industry, unallocated accounts are necessary for settlement of transactions which facilitate the operation of the value chain. For example, for a client looking to move some physical metal from a Swiss depository to Perth, we could arrange for the Swiss to give the Perth Mint unallocated into our London metals account (and they take possession/title to the physical) and we then give the client physical in Perth. This avoids the cost of shipment. But for the small to large investor who is more of a buy and hold and not interested in fancy trading, the unallocated probably is inferior.


Extrapolating old coin shop demand to a larger scale is highly idiotic, and I hope people following AD's comments can realize that.

Nickelsaver said...

"I am talking bout institutional and CB money - not retail/coin shops"

And this is precisely why Gold and only Gold is the loaded focal point. Copper, silver, nickel, platinum, etc., will not counter-balance fiat devaluation, because they will not use them.

Tommy2Tone said...

AD,
Greets!

Anonymous said...

Zero-Hedge have discovered some advertizing material of the FRBNY. That's not remarkable, but I thought the following quote was fun: former Chairman of the Federal Reserve’s Board of Governors, William McChesney Martin, praised gold as "a beautiful and noble metal. What is barbarous," Martin said, "is man’s enslavement to gold for monetary purposes."

Victor

AdvocatusDiaboli said...

Texan,
I guess it depends what kind of "paper gold" you buy. There are plenty of different ones. That's basically it.

Comme on, do you seriously think that everybody else outside the fanatic "physical gold bug world" is a complete moron and uncapable to read the broschure (and the fine print version), especially the really wealthy buyers?

Just for the fun, I just read the broschure of one of Germanys most favorite paper gold "products" WKNA0S9GB (XetraGold). It has a clear chapter about RISK and what you own (in this case obviously a claim(bond) on stored allocated+unallocated (the ratio is left open) gold in term of weight, which not you own, but the company, as long you dont take delivery).
It does not appear to be a pure derivative on gold. Would I buy it? No. Do other buy it? Yes.

Just look at the world of fiat currency. You want to own your "money"? Or you want to have a derivative on it? That's the difference between having the "original paper bill" or having it in your bank account. For some people it makes a difference (e.g. thinking of Greeks:D) for others it doesnt. (and yes, I know, maybe this is the most perfect example approach for the opposite that "paper suppresses", but just to show how different people are, they dont want the original, they dont want to take unconfiniences!!!)

So to conclude: Everybody gets what he wants (and especially deserves), no reason to say "paper suppresses gold", because also due to the variety of different types of paper gold (and especially their origin/design) this is not true (as I said earlier, excluding plain pure fraud).
Greets, AD

AdvocatusDiaboli said...

e_r,
thanks for the link, I guess Bron says it all most perfectly.
Greets, AD

Victory said...
This comment has been removed by the author.
Aquilus said...

Just posting this for newcomers to this blog:

Before you take any advice from AD, have a look at this gem:

"Has it been bought, by whom from whom, because you can only buy it at that price if there is a seller at that price (and is there a seller at those supposingly "small" prices...)? Has it been called for delivery? Or (who) accepted cash settlement? On the other hand others argue, that it is impossible to buy larger quantities... Can somebody from the FG-advocates finally make up his mind? In this context, I find this word "demand" kind of funny...

First, let me say: WOW! There's confusion in this person's mind if I ever saw one...

Point being: one should know some basics of gold market operation before crowing high and mighty. Since I'm not going to attempt to explain the LBMA operations here. But for newcomers that would rather learn the real deal about unalllocated and fractional reserve bullion banking, here are some links:

The View: A Classic Bank Run

Red Baron's LBMA EXPOSÉ

and also, the blog of Bron from the Perth Mint explaining how the current gold market works. It's fascinating, and first hand accounts from someone in the business: Bron Suchecki - How the industry works <- there are 43 articles in this series.


Oh and I cannot resist this other gem:

"So to conclude: Everybody gets what he wants (and especially deserves), no reason to say "paper suppresses gold", because also due to the variety of different types of paper gold (and especially their origin/design) this is not true (as I said earlier, excluding plain pure fraud)."

Newcomers may want to read why All paper is still a short position on gold to get a well thought analysis contradicting the gem above.

Tommy2Tone said...

By Gem, you mean like when I get up from the toilet and admire the handiwork, no?

Tommy2Tone said...

XD ! sorry for the gross humor. I couldn't resist. AD does that to me.

Texan said...

AD,

Ok I get your point. I think where you and I differ is "how much" of paper gold demand "thinks it owns gold" (and/or thinks it can be "faster than the other guy" and take delivery before system collapses).

There is obviously no way to answer that, but it cannot be zero. Or even approaching zero, IMO. Ergo gold has benn fractionally sold, like time shares to the same London apartment.

Hope they don't all decide to visit at the same time.

burningfiat said...

Paper promise holders, enjoy it while it lasts, cause when the levee breaks...... It's Rock'n'Roll time for FOFOA readers!

AdvocatusDiaboli said...

Texan,
yes I agree, let's look at the buyers/holders: Any kind of institutional buyer or fond does not f*cking care. That's not their job, that's not what they get paid for. They exclusively look at the nominal return what's on the supposingly balance sheet they can present, afterwards everybody is happy or....maybe not, if paper burns but that's the other storry.

So we have two option: In the end all paper burns or no paper burns. Option two is easy, everythings stays like it is.
Option one is the interesting one, when everything burns. Now in that case how does it feel like in real life? I guess it will be epic, because it will not just be a "small MF accounting problem". When that happens, I assume that it will have dramatic change on the real physical and social plane, "The day money dies". Forget the token monetary plane for a second, we will be talking about the real stuff you can eat.
Not nice. So in that case, what will be the "purchasing power" of gold? Who knows, nobody can predict even a realistic market place for that scenario, if we are lucky, best case it will be the same for gold as today.
I know, nobody wants to hear at the FG blog, because most of the people just want everything to stay the way it is, except to "be finally rich" just by themself, but that's why I say, have your Ferrari today and have fun, actually does not matter if option one or two plays out.
Again IMHO, Ari said the wisest word: Live your life and save the rest in ....
Oh, I just thought of it, option three: Everything stays the same, except just due to some legislation the paper gold holders will be left holding the bag. New game, new luck. The crazy GATA/FG assumption is somehow, that in this case gold buyers (the earlier paper holders?) are now willing to spend 50K on an ounce, that is 1-3K to mine.
What is your take: Option 1,2,3?
Greets, AD

«Oldest ‹Older   401 – 600 of 616   Newer› Newest»

Post a Comment

Comments are set on moderate, so they may or may not get through.